[obm-l] Re: [obm-l] ensino de matemática

2018-07-11 Por tôpico Leandro Martins
Caros,

Também tenho interesse em participar de tal discussão. Maior que minha
aproximação com a Matemática Olímpica, é minha aproximação com a
Matemática. Ainda maior é a aproximação de muitos alunos, sob diversos
aspectos.

Vejamos no que dá...

Abraço!

Em 11 de julho de 2018 12:30, Claudio Buffara 
escreveu:

> Prezados colegas da lista:
>
> Entendo que o tema pode ser off-topic pois não trata especificamente de
> problemas olímpicos, mas aqui vai de qualquer forma...
>
> Algum de vocês se interessa pelo ensino de matemática (escolar ou
> universitário)?
>
> Pergunto porque há anos tenho pensado na melhor forma de ensinar
> matemática (principalmente em termos de composição do currículo e de
> apresentação dos tópicos nos livros didáticos), estou convencido de que não
> estamos fazendo certo, nem na escola e nem na universidade, e gostaria de
> ter gente interessada pra debater idéias e, quem sabe, elaborar algum
> projeto mais concreto.
>
> Em linhas gerais, discordo da ordem em que os assuntos são abordados, na
> maioria dos livros.
> O foco é muito mais na ordem lógica (seguindo o rigor do método
> axiomático, mesmo em livros pra ensino médio) sem nenhuma preocupação:
> - com a motivação para os resultados que são apresentados (e, nos ensinos
> fundamental e médio, quase nunca demonstrados);
> - com tornar estes resultados intuitivos para o estudante.
>
> Também acho que certos assuntos deveriam ser incluídos e outros excluídos
> do currículo, mas este, pra mim, é um problema menor. Pois, qualquer que
> seja o tópico, se for bem ensinado e incentivar o aluno a pensar, já tá
> valendo.
>
> A meu ver, seria ideal se cada tópico do currículo de matemática fosse
> apresentado seguindo a sequência:
> identificação de padrões ("patterns") ==> formulação de conjecturas ==>
> demonstração destas conjecturas.
> Pois esta é a maneira como a matemática é criada.
> Mas acho que muito poucos professores estão capacitados pra ensinar
> matemática deste jeito.
>
> Em particular, no Ensino Médio, a ênfase nos últimos anos tem sido na tal
> contextualização, que pode ser vista em todo o seu esplendor nas provas do
> Enem.
> O resultado disso me parece ser um retrocesso na formação matemática dos
> alunos e também a disseminação da mentalidade de que a única matemática que
> deve ser estudada é aquela que é usada no dia-a-dia dos cidadãos comuns.
>
> E, na universidade, a coisa não é muito melhor, mesmo num assunto que só é
> visto na graduação em matemática. a análise real.
> Vejam só:
> Os livros tratam da topologia da reta antes de conceitos tais como
> compacidade e conexidade se mostrarem realmente necessários (o que, de
> fato, só ocorre em dimensão > 1; na reta, quase tudo pode ser demonstrado
> com base em sequências e no método da bisseção, que são coisas bastante
> intuitivas, mas que quase nunca são usadas).
>
> Limites e continuidade podem ser introduzidos também com base em
> sequências, interpretando-se os epsilons como margens de erro em
> aproximação.
>
> Aliás, a noção de que análise nada mais é do que uma teoria de
> aproximações quase nunca é mencionada.
> Por exemplo, foi só estudando a análise do R^n é que eu me dei conta de
> que a derivada é uma aproximação de uma função arbitrária por uma função
> afim.
> Antes disso, eu só sabia que "derivada = inclinação da reta tangente".
>
> Os livros também mencionam critérios de convergência de séries (Dirichlet,
> Abel, etc.) que vêm do nada (pois foram inventados para o estudo de séries
> de Fourier, que estes liros não abordam).
>
> E o principal resultado sobre convergência de séries de potências decorre
> quase trivialmente do estudo das PGs infinitas (assunto de Ensino Médio).
> Mas qual livro deixa isso explícito?
>
> E, pra terminar, poucos têm uma figura para ilustrar o teorema fundamental
> do cálculo que, com uma figura bem feita, fica bem intuitivo. No entanto, a
> análise na reta em geral é apresentada com um caráter aritmético/algébrico,
> mas quase nunca geométrico.
>
> Obrigado pela atenção.
>
> []s,
> Claudio.
>
> --
> Esta mensagem foi verificada pelo sistema de antivírus e
> acredita-se estar livre de perigo.

-- 
Esta mensagem foi verificada pelo sistema de antiv�rus e
 acredita-se estar livre de perigo.



Re: [obm-l] Ainda 24 divide mn + 1

2018-02-19 Por tôpico Leandro Martins
Além do 24, verifica-se que qualquer um de seus divisores também satisfazem
a implicação discutida, por conta dos fatores primos que compõem o 24.

Ou seja: 1 (trivial), 2, 3, 4, 6, 8 e 12.


Livre
de vírus. www.avast.com
.
<#m_2225411136329736553_DAB4FAD8-2DD7-40BB-A1B8-4E2AA1F9FDF2>

Em 15 de fevereiro de 2018 21:54, marcone augusto araújo borges <
marconeborge...@hotmail.com> escreveu:

> Que outros números, além do 24 satisfazem...
>
> 24 divide mn + 1  =
> >
> 24 divide m+n?
>
> --
> Esta mensagem foi verificada pelo sistema de antivírus e
> acredita-se estar livre de perigo.
>

-- 
Esta mensagem foi verificada pelo sistema de antiv�rus e
 acredita-se estar livre de perigo.



[obm-l] Re: [obm-l] Re: [obm-l] Re: [obm-l] Duas questões de matemática.

2016-08-10 Por tôpico Leandro Martins
Olá, amigos!

Quanto à questão filosófica: sabe-se que a soma  dos ângulos internos de um
triângulo, na geometria euclidiana plana, resulta 180 graus. Mas tais
ângulos não são definidos como suplementares.

Teríamos, aqui, uma pista de resposta negativa à questão de Douglas?

Abraço,

Leandro

Em 10/08/2016 13:50, "Pedro José"  escreveu:

> Boa tarde!
>
> Peguei um livro antigo do ginásio e a definição que lá consta é para dois
> ângulos.
> Mas como as coisas mudam. Pesquisei em sítios do Brasil, EUA e França,
> todas as definições são para dois ângulos.
> Já que se está falando em definições, quando estudava Análise no
> científico, Z+  incluía o 0 e Z+*  não incluía o 0, se chamavam, inteiros
> positivos e inteiros estritamente positivos (o asterisco simbolizava a
> exclusão do 0).
> Atualmente Z+ já exclui o 0.
> Alguém saberia dizer, quando e o porquê da mudança?
>
> Saudações,
> PJMS
>
> Em 8 de agosto de 2016 18:53, Rogerio Ponce  escreveu:
>
>> Ola' Douglas, a questao me parece perfeita.
>> Como as opcoes de resposta sao positivas, queremos a menor quantidade de
>> derrotas (ou seja, a maior quantidade de vitorias), que leve ao mesmo total
>> de premios.
>> Portanto, estamos falando das derrotas de maior valor (foram as 4
>> ultimas), acompanhadas por uma com o valor necessario para completar a soma
>> total.
>> Se considerassemos as 5 ultimas derrotas, o valor total seria
>> ultrapassado.
>> So' pode ser a letra "E".
>> []'s
>> Rogerio Ponce
>>
>> 2016-08-08 16:45 GMT-03:00 Douglas Oliveira de Lima <
>> profdouglaso.del...@gmail.com>:
>>
>>> Olá amigos, gostaria de uma ajuda em uma filosofia e uma questão.
>>>
>>> 1)Na definição de ângulos suplementares, seria para dois ângulos ou pode
>>> ser para mais de dois?
>>>
>>> 2)(Essa questão gostaria de saber se está mal elaborada) Carlos e
>>> Ricardo disputaram 15 partidas de boliche e ao fim de cada partida o
>>> perdedor pagava um prêmio em dinheiro para o vencedor. O prêmio para a
>>> primeira partida foi R$ 15,00 e o prê- mio de cada partida seguinte foi R$
>>> 5,00 a mais do que o valor da partida anterior. Ao final da disputa, ambos
>>> receberam o mesmo valor em dinheiro e nenhuma partida terminou empatada.
>>> Nes- sas condições, a maior diferença possível entre as vitórias e as
>>> derrotas de Ricardo é
>>> (A) 4. (B) 3. (C) 7. (D) 6. (E) 5.
>>>
>>> Att: Douglas Oliveira.
>>>
>>> --
>>> Esta mensagem foi verificada pelo sistema de antivírus e
>>> acredita-se estar livre de perigo.
>>
>>
>>
>> --
>> Esta mensagem foi verificada pelo sistema de antivírus e
>> acredita-se estar livre de perigo.
>>
>
>
> --
> Esta mensagem foi verificada pelo sistema de antivírus e
> acredita-se estar livre de perigo.

-- 
Esta mensagem foi verificada pelo sistema de antiv�rus e
 acredita-se estar livre de perigo.



[obm-l] Re: [obm-l] Re: [obm-l] Re: [obm-l] Re: [obm-l] Re: [obm-l] Re: [obm-l] Re: [obm-l] 2016 figurinhas e o número de retângulos de dimensões diferentes

2016-06-12 Por tôpico Leandro Martins
Boa tarde!

É uma honra, poder ajudar!

Grande abraço!
Em 11/06/2016 08:33, "Marcelo Gomes"  escreveu:

> Olá professor Leandro, bom dia.
>
> Muito obrigado pelo cuidado nas explicações e por suas preciosas
> orientações. Uma explicação como a que o senhor deu, é como acender uma
> lâmpada em local escuro!
>
> Muito obrigado!
>
> Abraços, Marcelo.
>
>
> Em 5 de junho de 2016 23:38, Leandro Martins 
> escreveu:
>
>> Boa tarde a todos!
>>
>> Grande Marcelo! Igualmente grato eu me sinto, pelos questionamentos.
>>
>> Em tua última tréplica, ficou claro pra mim que o retângulo 2016x1 foi
>> preservado. Dele, os retângulos considerados são construções parciais.
>>
>> De toda forma, por serem parciais, não utilizam integralmente as 2016
>> figurinhas. São 2016 figurinhas entre 63 triângulos.
>>
>> Teu raciocínio foi interessante: ao passo da existência de uma P.A. de
>> a[1] = r = 1, me inspirei em encontrar outra série que fosse uma partição
>> de 2016: aquela obtida da P. G. de a[1] = 32 e q = 2. Existem outras
>> partições arbitrárias de 2016, como podes constatar. Mas descaracterizam o
>> enunciado.
>>
>> Este problema, na verdade, tem como temática a Teoria dos Números. A
>> motivação na Geometria o torna ainda mais desafiador.
>>
>> Abraço!
>>
>> Leandro
>> Olá professor Leandro, muito obrigado mais uma vez, por seus preciosos
>> comentários e explicações!
>>
>> Desta forma como o senhor expressou: "Os retângulos formados por
>> Clarinha possuem a mesma área, por utilizarem todas as figurinhas",
>> consegui compreender...e desta forma concluí que era isto que o enunciado
>> tentou dizer.
>>
>> A questão dos "buracos", foi que eu não entendi. Na minha cabeça, não há
>> buracos entre os retângulos, todos os 2016, estão postos lado a lado, como
>> se fossem azulejos na parede, em uma grande e única linha.
>>
>> Abração e muito obrigado pelas ajudas!
>>
>> Marcelo.
>>
>>
>>
>> Em 30 de maio de 2016 18:26, Leandro Martins 
>> escreveu:
>>
>>> Boa tarde, pessoal!
>>>
>>> Caro prof. Marcelo, a soma dos termos da P. A. dada se encaixa como uma
>>> luva! Entretanto, os retângulos formados estariam com buracos entre si,
>>> contrariando o enunciado.
>>>
>>> Sinônimo de figurinhas arrumadas sem sobreposição ou buracos: figurinhas
>>> justapostas. Assim já vi em outro enunciado.
>>>
>>> Em tempo: na solução que enviei, onde se lê: "Os retângulos formados por
>>> Clarinha possuem a mesma área, por serem todos iguais", deve ser
>>> substituído por: "Os retângulos formados por Clarinha possuem a mesma área,
>>> por utilizarem todas as figurinhas."
>>>
>>> Grande abraço!
>>>
>>> Leandro
>>> Em 30/05/2016 07:32, "Marcelo Gomes"  escreveu:
>>>
>>>> Olá a todos, bom dia.
>>>>
>>>> Caro professor Leandro, muito obrigado pela ajuda! Não havia pensado
>>>> deste jeito. Obrigado por esclarecer.
>>>>
>>>> Em uma abordagem por Soma da PA, eu fiquei achando, que também cumpri
>>>> as exigências do enunciado da questão:
>>>>
>>>> 1- Em meu pensamento, pus todas as 2016 figurinhas lado a lado em uma
>>>> grande linha (1x2016=2016 u.a. para este retângulo)
>>>>
>>>> 2- Usei todas as figurinhas: 1º ret = 1 u.a. / 2º ret = 2 u.a. / 3º ret
>>>> = 3 u.a. ...63º ret = 63 u.a. (somando-se as parcelas temos 63 retângulos
>>>> de dimensões diferentes e 2016 figurinhas utilizadas).
>>>>
>>>> Estaria errado este pensamento que tive, em razão do enunciado
>>>> apresentado ?
>>>>
>>>> Abraços e muito obrigado pela ajuda e pelas explicações.
>>>>
>>>> Marcelo.
>>>>
>>>>
>>>> Em 29 de maio de 2016 22:56, Leandro Martins 
>>>> escreveu:
>>>>
>>>>> Caros, boa noite!
>>>>>
>>>>> Os retângulos formados por Clarinha possuem a mesma área, por serem
>>>>> todos iguais. Cada figurinha (quadrada) tem 1 u.a. (unidade de área).
>>>>> Utilizando todas as figurinhas, sabemos que o retângulo formado tem 2016
>>>>> u.a.
>>>>>
>>>>> O problema equivale a saber quantas são as multiplicações entre dois
>>>>> fatores (respectivamente, a base e a altura do retângulo formado) que
>>>>

[obm-l] Re: [obm-l] Re: [obm-l] Re: [obm-l] Re: [obm-l] Re: [obm-l] 2016 figurinhas e o número de retângulos de dimensões diferentes

2016-06-05 Por tôpico Leandro Martins
Boa tarde a todos!

Grande Marcelo! Igualmente grato eu me sinto, pelos questionamentos.

Em tua última tréplica, ficou claro pra mim que o retângulo 2016x1 foi
preservado. Dele, os retângulos considerados são construções parciais.

De toda forma, por serem parciais, não utilizam integralmente as 2016
figurinhas. São 2016 figurinhas entre 63 triângulos.

Teu raciocínio foi interessante: ao passo da existência de uma P.A. de a[1]
= r = 1, me inspirei em encontrar outra série que fosse uma partição de
2016: aquela obtida da P. G. de a[1] = 32 e q = 2. Existem outras partições
arbitrárias de 2016, como podes constatar. Mas descaracterizam o enunciado.

Este problema, na verdade, tem como temática a Teoria dos Números. A
motivação na Geometria o torna ainda mais desafiador.

Abraço!

Leandro
Olá professor Leandro, muito obrigado mais uma vez, por seus preciosos
comentários e explicações!

Desta forma como o senhor expressou: "Os retângulos formados por Clarinha
possuem a mesma área, por utilizarem todas as figurinhas", consegui
compreender...e desta forma concluí que era isto que o enunciado tentou
dizer.

A questão dos "buracos", foi que eu não entendi. Na minha cabeça, não há
buracos entre os retângulos, todos os 2016, estão postos lado a lado, como
se fossem azulejos na parede, em uma grande e única linha.

Abração e muito obrigado pelas ajudas!

Marcelo.



Em 30 de maio de 2016 18:26, Leandro Martins 
escreveu:

> Boa tarde, pessoal!
>
> Caro prof. Marcelo, a soma dos termos da P. A. dada se encaixa como uma
> luva! Entretanto, os retângulos formados estariam com buracos entre si,
> contrariando o enunciado.
>
> Sinônimo de figurinhas arrumadas sem sobreposição ou buracos: figurinhas
> justapostas. Assim já vi em outro enunciado.
>
> Em tempo: na solução que enviei, onde se lê: "Os retângulos formados por
> Clarinha possuem a mesma área, por serem todos iguais", deve ser
> substituído por: "Os retângulos formados por Clarinha possuem a mesma área,
> por utilizarem todas as figurinhas."
>
> Grande abraço!
>
> Leandro
> Em 30/05/2016 07:32, "Marcelo Gomes"  escreveu:
>
>> Olá a todos, bom dia.
>>
>> Caro professor Leandro, muito obrigado pela ajuda! Não havia pensado
>> deste jeito. Obrigado por esclarecer.
>>
>> Em uma abordagem por Soma da PA, eu fiquei achando, que também cumpri as
>> exigências do enunciado da questão:
>>
>> 1- Em meu pensamento, pus todas as 2016 figurinhas lado a lado em uma
>> grande linha (1x2016=2016 u.a. para este retângulo)
>>
>> 2- Usei todas as figurinhas: 1º ret = 1 u.a. / 2º ret = 2 u.a. / 3º ret =
>> 3 u.a. ...63º ret = 63 u.a. (somando-se as parcelas temos 63 retângulos de
>> dimensões diferentes e 2016 figurinhas utilizadas).
>>
>> Estaria errado este pensamento que tive, em razão do enunciado
>> apresentado ?
>>
>> Abraços e muito obrigado pela ajuda e pelas explicações.
>>
>> Marcelo.
>>
>>
>> Em 29 de maio de 2016 22:56, Leandro Martins 
>> escreveu:
>>
>>> Caros, boa noite!
>>>
>>> Os retângulos formados por Clarinha possuem a mesma área, por serem
>>> todos iguais. Cada figurinha (quadrada) tem 1 u.a. (unidade de área).
>>> Utilizando todas as figurinhas, sabemos que o retângulo formado tem 2016
>>> u.a.
>>>
>>> O problema equivale a saber quantas são as multiplicações entre dois
>>> fatores (respectivamente, a base e a altura do retângulo formado) que
>>> resultam em 2016.
>>>
>>> Temos que 2016 = 2^5.3^2.7, procedendo sua fatoração em primos. Daí
>>> calculamos que 2016 possui (5+1). (2+1). (1+1) = 36 divisores. Obtemos 2016
>>> pelo produto entre o divisor imediatamente menor e o divisor imediatamente
>>> maior (1x2016, 2x1008, ...) de 18 maneiras diferentes. Logo, são 18
>>> retângulos de dimensões diferentes formados com todas as figurinhas.
>>>
>>> Abraço!
>>>
>>> Leandro
>>> Em 28/05/2016 14:06, "Marcelo Gomes"  escreveu:
>>>
>>>> Olá a todos, boa tarde.
>>>>
>>>> Peço, o auxílio, de quem dispuser de um tempinho, para explicar o
>>>> porquê do gabarito desta questão ser 18.
>>>>
>>>> "Clarinha arruma 2016 figurinhas iguais, colocando-as lado a lado,
>>>> formando retângulos sem superposições ou buracos. O número de retângulos de
>>>> dimensões diferentes formados usando todas as figurinhas é: "
>>>>
>>>> (A) 14.
>>>>
>>>> (B) 18.
>>>>
>>>> (C) 21.
>>>>
>>>> (D) 24.
>>>>
>>>>

[obm-l] Re: [obm-l] Re: [obm-l] Re: [obm-l] 2016 figurinhas e o número de retângulos de dimensões diferentes

2016-05-30 Por tôpico Leandro Martins
Boa tarde, pessoal!

Caro prof. Marcelo, a soma dos termos da P. A. dada se encaixa como uma
luva! Entretanto, os retângulos formados estariam com buracos entre si,
contrariando o enunciado.

Sinônimo de figurinhas arrumadas sem sobreposição ou buracos: figurinhas
justapostas. Assim já vi em outro enunciado.

Em tempo: na solução que enviei, onde se lê: "Os retângulos formados por
Clarinha possuem a mesma área, por serem todos iguais", deve ser
substituído por: "Os retângulos formados por Clarinha possuem a mesma área,
por utilizarem todas as figurinhas."

Grande abraço!

Leandro
Em 30/05/2016 07:32, "Marcelo Gomes"  escreveu:

> Olá a todos, bom dia.
>
> Caro professor Leandro, muito obrigado pela ajuda! Não havia pensado deste
> jeito. Obrigado por esclarecer.
>
> Em uma abordagem por Soma da PA, eu fiquei achando, que também cumpri as
> exigências do enunciado da questão:
>
> 1- Em meu pensamento, pus todas as 2016 figurinhas lado a lado em uma
> grande linha (1x2016=2016 u.a. para este retângulo)
>
> 2- Usei todas as figurinhas: 1º ret = 1 u.a. / 2º ret = 2 u.a. / 3º ret =
> 3 u.a. ...63º ret = 63 u.a. (somando-se as parcelas temos 63 retângulos de
> dimensões diferentes e 2016 figurinhas utilizadas).
>
> Estaria errado este pensamento que tive, em razão do enunciado apresentado
> ?
>
> Abraços e muito obrigado pela ajuda e pelas explicações.
>
> Marcelo.
>
>
> Em 29 de maio de 2016 22:56, Leandro Martins 
> escreveu:
>
>> Caros, boa noite!
>>
>> Os retângulos formados por Clarinha possuem a mesma área, por serem todos
>> iguais. Cada figurinha (quadrada) tem 1 u.a. (unidade de área). Utilizando
>> todas as figurinhas, sabemos que o retângulo formado tem 2016 u.a.
>>
>> O problema equivale a saber quantas são as multiplicações entre dois
>> fatores (respectivamente, a base e a altura do retângulo formado) que
>> resultam em 2016.
>>
>> Temos que 2016 = 2^5.3^2.7, procedendo sua fatoração em primos. Daí
>> calculamos que 2016 possui (5+1). (2+1). (1+1) = 36 divisores. Obtemos 2016
>> pelo produto entre o divisor imediatamente menor e o divisor imediatamente
>> maior (1x2016, 2x1008, ...) de 18 maneiras diferentes. Logo, são 18
>> retângulos de dimensões diferentes formados com todas as figurinhas.
>>
>> Abraço!
>>
>> Leandro
>> Em 28/05/2016 14:06, "Marcelo Gomes"  escreveu:
>>
>>> Olá a todos, boa tarde.
>>>
>>> Peço, o auxílio, de quem dispuser de um tempinho, para explicar o porquê
>>> do gabarito desta questão ser 18.
>>>
>>> "Clarinha arruma 2016 figurinhas iguais, colocando-as lado a lado,
>>> formando retângulos sem superposições ou buracos. O número de retângulos de
>>> dimensões diferentes formados usando todas as figurinhas é: "
>>>
>>> (A) 14.
>>>
>>> (B) 18.
>>>
>>> (C) 21.
>>>
>>> (D) 24.
>>>
>>>(E) 35.
>>> Não consegui montar um cálculo que chegasse neste valor. Tentei por soma
>>> de PA, considerando razão 1 e encontrei an = n = 63.
>>>
>>> Abraços, Marcelo.
>>>
>>> --
>>> Esta mensagem foi verificada pelo sistema de antivírus e
>>> acredita-se estar livre de perigo.
>>
>>
>> --
>> Esta mensagem foi verificada pelo sistema de antivírus e
>> acredita-se estar livre de perigo.
>
>
>
> --
> Esta mensagem foi verificada pelo sistema de antivírus e
> acredita-se estar livre de perigo.

-- 
Esta mensagem foi verificada pelo sistema de antiv�rus e
 acredita-se estar livre de perigo.



[obm-l] Re: [obm-l] 2016 figurinhas e o número de retângulos de dimensões diferentes

2016-05-29 Por tôpico Leandro Martins
Caros, boa noite!

Os retângulos formados por Clarinha possuem a mesma área, por serem todos
iguais. Cada figurinha (quadrada) tem 1 u.a. (unidade de área). Utilizando
todas as figurinhas, sabemos que o retângulo formado tem 2016 u.a.

O problema equivale a saber quantas são as multiplicações entre dois
fatores (respectivamente, a base e a altura do retângulo formado) que
resultam em 2016.

Temos que 2016 = 2^5.3^2.7, procedendo sua fatoração em primos. Daí
calculamos que 2016 possui (5+1). (2+1). (1+1) = 36 divisores. Obtemos 2016
pelo produto entre o divisor imediatamente menor e o divisor imediatamente
maior (1x2016, 2x1008, ...) de 18 maneiras diferentes. Logo, são 18
retângulos de dimensões diferentes formados com todas as figurinhas.

Abraço!

Leandro
Em 28/05/2016 14:06, "Marcelo Gomes"  escreveu:

> Olá a todos, boa tarde.
>
> Peço, o auxílio, de quem dispuser de um tempinho, para explicar o porquê
> do gabarito desta questão ser 18.
>
> "Clarinha arruma 2016 figurinhas iguais, colocando-as lado a lado,
> formando retângulos sem superposições ou buracos. O número de retângulos de
> dimensões diferentes formados usando todas as figurinhas é: "
>
> (A) 14.
>
> (B) 18.
>
> (C) 21.
>
> (D) 24.
>
>(E) 35.
> Não consegui montar um cálculo que chegasse neste valor. Tentei por soma
> de PA, considerando razão 1 e encontrei an = n = 63.
>
> Abraços, Marcelo.
>
> --
> Esta mensagem foi verificada pelo sistema de antivírus e
> acredita-se estar livre de perigo.

-- 
Esta mensagem foi verificada pelo sistema de antiv�rus e
 acredita-se estar livre de perigo.



Re: [obm-l] Derivada de um produto de funções

2015-09-13 Por tôpico LEANDRO L RECOVA
Procure pela formula de Leibniz.

Sent from my iPhone

> On Sep 13, 2015, at 9:05 PM, Israel Meireles Chrisostomo 
>  wrote:
> 
> A fórmula da derivada de um produto de funções vale quando se tem 
> infinitas funções?
> Isto é, vale que 
> d/dx(f_1(x)f_2(x)f_3(x)...f_n(x))=f '_1(x)f_2(x)f_3(x)...f_n(x)+f_1(x)f 
> '_2(x)f_3(x)...f_n(x)+
> f_1(x)f_2(x)f '_3(x)...f_n(x)+
> 
> -- 
> Esta mensagem foi verificada pelo sistema de antivírus e 
> acredita-se estar livre de perigo.

-- 
Esta mensagem foi verificada pelo sistema de antiv�rus e
 acredita-se estar livre de perigo.


=
Instru��es para entrar na lista, sair da lista e usar a lista em
http://www.mat.puc-rio.br/~obmlistas/obm-l.html
=


Re: [obm-l] Uma soma

2014-07-29 Por tôpico LEANDRO L RECOVA
Perdao, cexp(1)=1.

Sent from my iPhone

> On Jul 29, 2014, at 7:58 AM, "LEANDRO L RECOVA"  wrote:
> 
> Fatore o c e a soma e a serie de exp(x), dai fica
> 
> cexp(x)=1.
> 
> Sent from my iPhone
> 
>> On Jul 28, 2014, at 8:11 PM, "João Sousa"  wrote:
>> 
>> Pessoal se \sum_{k=0}^{+infty} \frac{c}{k!} = 1, qual é o valor de c, onde 
>> c é constante?
>> 
>> -- 
>> Esta mensagem foi verificada pelo sistema de antivírus e 
>> acredita-se estar livre de perigo.
> 
> -- 
> Esta mensagem foi verificada pelo sistema de antivírus e 
> acredita-se estar livre de perigo.

-- 
Esta mensagem foi verificada pelo sistema de antiv�rus e
 acredita-se estar livre de perigo.



Re: [obm-l] Uma soma

2014-07-29 Por tôpico LEANDRO L RECOVA
Fatore o c e a soma e a serie de exp(x), dai fica

cexp(x)=1.

Sent from my iPhone

> On Jul 28, 2014, at 8:11 PM, "João Sousa"  wrote:
> 
> Pessoal se \sum_{k=0}^{+infty} \frac{c}{k!} = 1, qual é o valor de c, onde c 
> é constante?
> 
> -- 
> Esta mensagem foi verificada pelo sistema de antivírus e 
> acredita-se estar livre de perigo.

-- 
Esta mensagem foi verificada pelo sistema de antiv�rus e
 acredita-se estar livre de perigo.



Re: [obm-l] Isomorfismo

2014-05-28 Por tôpico LEANDRO L RECOVA
Voce pode representar os numeros complexos na forma polar. Use a representacao 
de Euler. 

Sent from my iPhone

> On May 28, 2014, at 9:29 AM, "Kleber Santana" <1kleb...@gmail.com> wrote:
> 
> G={2^m*2^n|m,n Z }, J={m+ni| m,n Z}
> 
> Mostrei que f:J--->G é homomorfismo, é injetora, mas não consigo provar 
> que é sobrejetora.
> 
> Alguém pode ajudar?
> 
> Grato,
> 
> Kleber.
> 
> -- 
> Esta mensagem foi verificada pelo sistema de antivírus e 
> acredita-se estar livre de perigo.

-- 
Esta mensagem foi verificada pelo sistema de antiv�rus e
 acredita-se estar livre de perigo.


=
Instru��es para entrar na lista, sair da lista e usar a lista em
http://www.mat.puc-rio.br/~obmlistas/obm-l.html
=


RE: [obm-l] off topic - livro caronnet

2014-04-14 Por tôpico Anderson Leandro Zulin
Olá Regis, se possível, passa o link para mim.alzu...@hotmail.com

Eu tenho alguns livros aquihttp://educzulin.blogspot.com.br/
E tenho muito mais no meu not.Se estiverem a procura de algum, me escrevam.

Date: Mon, 14 Apr 2014 13:35:45 -0700
From: regisgbar...@yahoo.com.br
Subject: Re: [obm-l] off topic - livro caronnet
To: obm-l@mat.puc-rio.br

Olá PessoalNo ano passado teve contato com todos os livros do caronnet e fiz um 
scan deles caso vocês queiram é só pedir que mando link para o email pessoal.
Regis Em Segunda-feira, 14 de Abril de 2014 14:34, Sergio Lima 
 escreveu:
oi Hermann,
Procure na Estante Virtual. Acho que voce encontrarah lah.Os livros tem 
excelentes exercicios.
Abraco,sergio


2014-04-20 13:06 GMT-03:00 Hermann :








OFF - TOPIC
 
Meus amigos, gostaria de saber se alguém já viu os 
livros do caronnet volumes 1, 2, 4 e 5.
 
Porque há anos eu procuro e nunca vi.
 
E a coleção é boa, na opinião de 
vocês?
 
abraços
Hermann
--

Esta mensagem foi verificada pelo sistema de antivírus e 

 acredita-se estar livre de perigo.




--

Esta mensagem foi verificada pelo sistema de antivírus e 

 acredita-se estar livre de perigo.


  
--

Esta mensagem foi verificada pelo sistema de antivírus e 

 acredita-se estar livre de perigo.   
-- 
Esta mensagem foi verificada pelo sistema de antivírus e
 acredita-se estar livre de perigo.



Re: [obm-l] Vetores

2013-11-21 Por tôpico LEANDRO L RECOVA
Voce tem que tomar o conjugado complexo. 

Sent from my iPhone

> On Nov 21, 2013, at 8:11 AM, "Athos Cotta Couto"  
> wrote:
> 
> Olá.
> 
> Para dois vetores u e v serem iguais em um espaço vetorial real E, basta que:
>  = 
> Para todo x em E (ou para todo x de uma base de E).
> 
> Agora, tomando um espaço complexo, gostaria de saber se a condição:
> + = +
> É suficiente para falarmos que u = v. Se sim, por que? Se não, há uma 
> semelhante?
> 
> Obrigado pela ajuda.
> 
> 
> 
> -- 
> Esta mensagem foi verificada pelo sistema de antivírus e 
> acredita-se estar livre de perigo.

-- 
Esta mensagem foi verificada pelo sistema de antiv�rus e
 acredita-se estar livre de perigo.


=
Instru��es para entrar na lista, sair da lista e usar a lista em
http://www.mat.puc-rio.br/~obmlistas/obm-l.html
=


[obm-l] RE: [obm-l] RE: [obm-l] Re: [obm-l] Função periódica

2013-06-24 Por tôpico LEANDRO L RECOVA
Seja I=[0,T] o intervalo em que f:R->R e periodica. Como f e continua e 
definida sobre um conjunto compacto, entao f admite maximo e minimo. 

From: marconeborge...@hotmail.com
To: obm-l@mat.puc-rio.br
Subject: [obm-l] RE: [obm-l] Re: [obm-l] Função periódica 
Date: Mon, 24 Jun 2013 15:30:13 +




valeu,Saulo!

Date: Sun, 23 Jun 2013 18:27:20 -0300
Subject: [obm-l] Re: [obm-l] Função periódica
From: saulo.nil...@gmail.com
To: obm-l@mat.puc-rio.br

procurando x1  f(x1)=0, se x1 e raiz entao 
x1+p tambem e logo o grafico da funçao corta o eixo x em dois pontos tendo um 
maximo ou um minimo.



2013/5/1 marcone augusto araújo borges 




Uma função f:R->R é dita periódica quando existe um número real p > 0,tal que 
f(x) = f(x + p),para
todo x real.Prove que toda função periódica continua admite máximo e admite 
mínimo
  




--

Esta mensagem foi verificada pelo sistema de antivírus e 

 acredita-se estar livre de perigo.   
--

Esta mensagem foi verificada pelo sistema de antivírus e 

 acredita-se estar livre de perigo.   
-- 
Esta mensagem foi verificada pelo sistema de antivírus e
 acredita-se estar livre de perigo.



[obm-l] RE: [obm-l] Re: [obm-l] RE: [obm-l] Equação Funcional ou Recorrência

2013-04-01 Por tôpico LEANDRO L RECOVA
Ok. Eu pensei que na definicao de R^{+}, o zero estava excluido. Nao percebi.
 Date: Sun, 31 Mar 2013 18:51:26 -0300
Subject: [obm-l] Re: [obm-l] RE: [obm-l] Equação Funcional ou Recorrência
From: pedromatematic...@gmail.com
To: obm-l@mat.puc-rio.br

Olá Leandro, consegui resolver o problema  e muito obrigado pela sugestão.
Seguinte: 


Faça x = 0  ==> f(f(0)) + af(0) = b(a+b) 0 = 0 ==> f(f(0)) = - af(0)


Seja f(0) = y_1 ==> f(f(0)) = f(y_1) = - a f(0)
Agora faça f(y_1) = y_2

perceba a recorrência: y_n = f(y_(n-1)). Substituindo na equação funcional 
temos:

y_(n+2) +a y_(n+1) -b(a+b) y_n = 0


Eq. Característica: r^2 +ar - b(a+b) = 0

Daí continua...
Abç


Em 31 de março de 2013 16:48, LEANDRO L RECOVA  escreveu:




Eu pensei no seguinte:
y=f(x). Entao,
f(y) + ay = b(a+b)x 
f(y) = b(a+b)x-ay 
Ja que f assume valores reais positivos (R^+), entao, temos que ter f(y) > 0, 
ou seja, 

ay < b(a+b)x  => f(x) < b/a (a+b)x.  (*)
As funcoes f devem satisfazer a condicao (*).  Vou continuar pensando na 
questao.

Date: Sat, 30 Mar 2013 16:09:29 -0300
Subject: [obm-l] Equação Funcional ou Recorrência
From: pedromatematic...@gmail.com
To: obm-l@mat.puc-rio.br


Essa questão estava numa lista de Equações Recorrentes. É possível resolvê-la 
por recorrência?

Ache todas as funções f: R^+ -->R^+ tais que f(f(x)) + af(x) = b(a+b)x onde a,b 
\in R^+.

-- 


Pedro Jerônimo S. de O.
Júnior


Professor
de Matemática


Geo João Pessoa
– PB 




--

Esta mensagem foi verificada pelo sistema de antivírus e 

 acredita-se estar livre de perigo.   
--

Esta mensagem foi verificada pelo sistema de antivírus e 

 acredita-se estar livre de perigo.




-- 


Pedro Jerônimo S. de O.
Júnior

Professor
de Matemática

Geo João Pessoa
– PB 




--

Esta mensagem foi verificada pelo sistema de antivírus e 

 acredita-se estar livre de perigo.   
-- 
Esta mensagem foi verificada pelo sistema de antivírus e
 acredita-se estar livre de perigo.



[obm-l] RE: [obm-l] Equação Funcional ou Recorrência

2013-03-31 Por tôpico LEANDRO L RECOVA
Eu pensei no seguinte:
y=f(x). Entao,
f(y) + ay = b(a+b)x 
f(y) = b(a+b)x-ay 
Ja que f assume valores reais positivos (R^+), entao, temos que ter f(y) > 0, 
ou seja, 
ay < b(a+b)x  => f(x) < b/a (a+b)x.  (*)
As funcoes f devem satisfazer a condicao (*).  Vou continuar pensando na 
questao.
Date: Sat, 30 Mar 2013 16:09:29 -0300
Subject: [obm-l] Equação Funcional ou Recorrência
From: pedromatematic...@gmail.com
To: obm-l@mat.puc-rio.br

Essa questão estava numa lista de Equações Recorrentes. É possível resolvê-la 
por recorrência?

Ache todas as funções f: R^+ -->R^+ tais que f(f(x)) + af(x) = b(a+b)x onde a,b 
\in R^+.
-- 


Pedro Jerônimo S. de O.
Júnior

Professor
de Matemática

Geo João Pessoa
– PB 



--

Esta mensagem foi verificada pelo sistema de antivírus e 

 acredita-se estar livre de perigo.   
-- 
Esta mensagem foi verificada pelo sistema de antivírus e
 acredita-se estar livre de perigo.



[obm-l] RE: [obm-l] RE: [obm-l] RE: [obm-l] Homeomorfismo dessa função

2012-10-17 Por tôpico LEANDRO L RECOVA

Rafael,
Ou, calcule diretamente a inversa considerando que voce ja provou a bijecao: 
f^-1: S^1\(0,1)-> (0,1). 
Se y esta em S1 entao e da forma y=(y1,y2)=(cos(2pi)t,sin(2pi)t), para t em 
(0,1).
y1=cos(2pi)ty2=sin(2pit)t
Divida y2/y1, e voce obtem que 
tan(2pi)t=y2/y1 
i.e,
t = atan (y2/y1), para todo y1,y2 em S^{1}\(0,1). E agora deixo contigo! 


Date: Tue, 16 Oct 2012 17:28:42 -0300
From: ar...@usp.br
To: obm-l@mat.puc-rio.br
Subject: Re: [obm-l] RE: [obm-l] RE: [obm-l] Homeomorfismo dessa função

   Seja I um intervalo aberto de (0,1). Não é difícil de ver que f(I) é um arco 
aberto do círculo. Como todo aberto de (0,1) é uma união enumerável de 
intervalos abertos segue-se que f é uma aplicação aberta. Sendo f contínua e 
sobrejetora (vc fez isto!) então f é um homeomorfismo.
   Veja se tá bom assim... 

Arlane Manoel S Silva
  Departamento de Matemática Aplicada
Instituto de Matemática e Estatística-USP

De: "Rafael Chavez" 
Para: obm-l@mat.puc-rio.br
Enviadas: Terça-feira, 16 de Outubro de 2012 16:47:20
Assunto: [obm-l] RE: [obm-l] RE: [obm-l] Homeomorfismo dessa função 





olá Leandro,
Eu tentei, mas não tive sucesso, achei mais complicado.
From: leandrorec...@msn.com
To: obm-l@mat.puc-rio.br
Subject: [obm-l] RE: [obm-l] Homeomorfismo dessa função
Date: Tue, 16 Oct 2012 12:10:48 -0700





Nao ha perguntas bobas.
Porque voce nao mostra que a imagem de todo aberto de f e aberto. Dai, voce 
prova A^-1 e continua.

From: matematico1...@hotmail.com
To: obm-l@mat.puc-rio.br
Subject: [obm-l] Homeomorfismo dessa função
Date: Mon, 15 Oct 2012 14:57:08 +0300





Olá pessoal,
Eu estou quebrando a cabeça para provar o homeomorfismo dessa 
função:f:(0,1)-->Círculo menos o ponto (0,1) definida por  
t--->(cos(2pi)t,sen(2pi)t)A continuidade é fácil, pois cada função componente é 
contínua, mas não consigo provar que a inversa é contínuaalguma luz?
Obrigado

  

  

[obm-l] RE: [obm-l] Homeomorfismo dessa função

2012-10-16 Por tôpico LEANDRO L RECOVA

Nao ha perguntas bobas.
Porque voce nao mostra que a imagem de todo aberto de f e aberto. Dai, voce 
prova A^-1 e continua.

From: matematico1...@hotmail.com
To: obm-l@mat.puc-rio.br
Subject: [obm-l] Homeomorfismo dessa função
Date: Mon, 15 Oct 2012 14:57:08 +0300





Olá pessoal,
Eu estou quebrando a cabeça para provar o homeomorfismo dessa 
função:f:(0,1)-->Círculo menos o ponto (0,1) definida por  
t--->(cos(2pi)t,sen(2pi)t)A continuidade é fácil, pois cada função componente é 
contínua, mas não consigo provar que a inversa é contínuaalguma luz?
Obrigado

  

RE: [obm-l] ajuda (faltou dizer que:)

2012-08-30 Por tôpico LEANDRO L RECOVA

Fazendo x=y=1,
 
f(1)^2 - f(1) -2 =0.
 
Equacao do 2o grau. 
 
Delta = 1 -4(-2) = 9
 
f(1) = (1 + 3)/2 ou
 
f(1) = (1-3)/2. Essa ultima esta descartada.
 
Entao, f(1)=2. 
 
Regards,
 



From: mat.mo...@gmail.com
Date: Thu, 30 Aug 2012 07:56:05 -0300
Subject: [obm-l] ajuda (faltou dizer que:)
To: obm-l@mat.puc-rio.br



Será que alguém poderia me ajudar na seguinte questão:

Seja f: R*+ - R*+ uma função tal que

f(x).f(y)-f(x.y)=x/y+y/x, então f(1)=?

  

RE: [obm-l] RE: [obm-l] Questão simples

2011-12-23 Por tôpico LEANDRO L RECOVA
Eu encontrei 13. O numero e dado por. Y=sqrt((n+1)n)*(n-1)! Sent from my HTC 
Touch Pro2 on the Now Network from Sprint®.


-Original Message-
From: LEANDRO L RECOVA
Sent: 12/23/2011 4:31:23 PM
To: obm-l@mat.puc-rio.br
Subject: [obm-l] RE: [obm-l] Questão simples
Marcone,

Escreva cada termo usando a fatoracao: (n^2-1)=(n+1)(n-1).

O resultado sai bem rapido.

Saudacoes,

Leandro Recova
Los Angeles, California.


From: marconeborge...@hotmail.com
To: obm-l@mat.puc-rio.br
Subject: [obm-l] Questão simples
Date: Fri, 23 Dec 2011 13:39:17 +

Qual é o menor natural n para o qual (2^2 - 1).(3^2 - 1).(4^2 -1)...(n^2 - 1) é 
um quadrado perfeito?

Como se diz aqui,essa eu fiz ??no braço``.Fui calculando cada fator e pareando 
os fatores iguais ou seus fatores primos.
Encontrei n=8.Mas deve haver solução mais interessante.Obrigado por qualquer 
esclarecimento,abraço.


[obm-l] RE: [obm-l] Questão simples

2011-12-23 Por tôpico LEANDRO L RECOVA

Marcone,
 
Escreva cada termo usando a fatoracao: (n^2-1)=(n+1)(n-1).
 
O resultado sai bem rapido.
 
Saudacoes,
 
Leandro Recova
Los Angeles, California.
 



From: marconeborge...@hotmail.com
To: obm-l@mat.puc-rio.br
Subject: [obm-l] Questão simples
Date: Fri, 23 Dec 2011 13:39:17 +




Qual é o menor natural n para o qual (2^2 - 1).(3^2 - 1).(4^2 -1)...(n^2 - 1) é 
um quadrado perfeito?
 
Como se diz aqui,essa eu fiz ´´no braço``.Fui calculando cada fator e pareando 
os fatores iguais ou seus fatores primos.
Encontrei n=8.Mas deve haver solução mais interessante.Obrigado por qualquer 
esclarecimento,abraço.
  

RE: [obm-l] Valor mínimo

2011-09-23 Por tôpico LEANDRO L RECOVA
Seja f:R->R tal que f(x)=asin x + bcos x. O ponto critico dessa equacao deve 
satisfazer f'(x)=0, isto e, acos x - bsin x=0 a^2(cos x)^2=b^2(sin x)^2 
(a^2+b^2)(cos x)^2=b^2 Resolva para cos x e obtenha sin x pela relacao 
fundamental. Substitua em f em e a resposta segue facilmente. Leandro Sent from 
my HTC Touch Pro2 on the Now Network from Sprint®.


-Original Message-
From: marcone augusto araújo borges
Sent: 9/23/2011 9:46:47 PM
To: obm-l@mat.puc-rio.br
Subject: [obm-l] Valor mínimo
Como provar que o valor minimo de asenx + bcosx = - raiz(a^2 + b^2) ?


Re: [obm-l] A procura de um livro! (off-topic)

2011-07-20 Por tôpico Leandro Lima
Olá, Pedro!

http://www.archive.org/details/117770259 

Espero que sirva para seus propósitos.

Abraço.

Leandro. 

From: Pedro Júnior 
Sent: Wednesday, July 20, 2011 8:13 AM
To: obm-l 
Subject: [obm-l] A procura de um livro! (off-topic)

Alguém poderia me indicar algum site que tenha o livro: 
L. E. Dickson, Algebras and their Arithmetics, University of Chicago Press, 1923
p.s.: poderia ser para download, pois pela data acho que não tem mais para 
vender!
-- 

Pedro Jerônimo S. de O. Júnior

Professor de Matemática

Geo João Pessoa – PB 



[obm-l] RE: [obm-l] Números Inteiros

2011-05-29 Por tôpico LEANDRO L RECOVA

Pedro,
 
Eu pensei assim: Seja x o numero que voce quer determinar. Ja que x tem dois 
algarismos, entao, x  e da forma ab:
 
x = 10a + b, com a,b numeros naturais com a entre 1 e 9 e b entre 0 e 9.
 
Eu fiquei em duvida na redacao da questao e entendi que que voce quer 
determinar a diferenca entre x e o produto dos algarismos a e b. Se nao for 
esse caso, me corrija.
 
Entao, queremos determinar o numero de inteiros positivos de dois algarismos 
tais que x-ab=12. Ou seja,
 
(10a + b) - ab = 12
 
Isolando a, temos: a=(12-b)/(10-b). 
 
Para que isso esteja bem definido temos que ter b < 10. Entao, voce tem que 
testar os numeros de 0 a 9 e ver quais te dao um valor de a inteiro. 
 
As possibilidades sao: b=8, a=2 portanto x=28, e b=9, a=3, portanto x=39. Dessa 
forma voce tem somente dois numeros que satisfazem a condicao do problema. 
 
Observe que 28-(8.2)=28-16=12 e 39-(9.3)=39-27=12. 
 
Saudacoes,
 
Leandro Recova
Los Angeles, EUA.

 


Date: Sun, 29 May 2011 09:35:00 -0300
Subject: [obm-l] Números Inteiros
From: pedromatematic...@gmail.com
To: obm-l@mat.puc-rio.br

10ª Questão da Olimpíada Campinense de Matemática - 2011 - Realizada em 28 de 
Maio de 2011.

10. Qual da quantidade de números inteiros positivos de dois algarismos tais 
que a diferença entre o número e o produto seja 12.
-- 

Pedro Jerônimo S. de O. Júnior
Professor de Matemática
Geo João Pessoa – PB 

  

RE: [obm-l] Números Inteiros

2011-05-29 Por tôpico LEANDRO L RECOVA
Pedro, A redacao da questao esta correta? O produto que voce se refere e o 
produto dos algarismos? Leandro Sent from my HTC Touch Pro2 on the Now Network 
from Sprint®.


-Original Message-
From: Pedro Júnior
Sent: 5/29/2011 12:35:00 PM
To: obm-l
Subject: [obm-l] Números Inteiros
10ª Questão da Olimpíada Campinense de Matemática - 2011 - Realizada em 28 de 
Maio de 2011.

10. Qual da quantidade de números inteiros positivos de dois algarismos tais 
que a diferença entre o número e o produto seja 12.

--

Pedro Jerônimo S. de O. Júnior

Professor de Matemática

Geo João Pessoa - PB



RE: [obm-l] Subtrair 2 de 3 ( 3 - 2 ou 2 - 3 ?)

2011-05-18 Por tôpico LEANDRO L RECOVA
O correto e 3-2 sem duvida. Isso e portugues. Sent from my HTC Touch Pro2 on 
the Now Network from Sprint®.


-Original Message-
From: Tiago
Sent: 5/18/2011 1:24:16 PM
To: obm-l@mat.puc-rio.br
Subject: Re: [obm-l] Subtrair 2 de 3 ( 3 - 2 ou 2 - 3 ?)
Creio que ambas interpretações podem estar corretas, por peculiaridades do 
português, mas não tenho certeza.

Mas se eu tivesse que escolher, escolheria 3 - 2, sem dúvidas.

2011/5/17 Paulo Argolo mailto:argolopa...@hotmail.com>>


Caros Colegas,


 Subtrair 2 de 3 significa calcular 3 - 2 ou 2 - 3?


Abraços!
Paulo
=
Instruções para entrar na lista, sair da lista e usar a lista em
http://www.mat.puc-rio.br/~obmlistas/obm-l.html
=



--
Tiago J. Fonseca
http://legauss.blogspot.com


[obm-l] RE: [obm-l] RES: [obm-l] Re: [obm-l] funçao de classe C^infinito

2011-02-11 Por tôpico LEANDRO L RECOVA
Que tal uma funcao trigonometrica f(x)=sin(x)). Sent from my HTC Touch Pro2 on 
the Now Network from Sprint®.


-Original Message-
From: Artur Costa Steiner
Sent: 2/11/2011 2:29:19 PM
To: obm-l@mat.puc-rio.br
Subject: [obm-l] RES: [obm-l] Re: [obm-l] funï¿œao de classe C^infinito
Esta funᅵᅵo nᅵo ᅵ de classe C^infinito. Nᅵo ᅵ derivᅵvel em 0


owner-ob...@mat.puc-rio.br [mailto:owner-ob...@mat.puc-rio.br] Em nome de Julio 
Cesar
Enviada em: sexta-feira, 11 de fevereiro de 2011 12:09
Para: obm-l@mat.puc-rio.br
Assunto: [obm-l] Re: [obm-l] funï¿œao de classe C^infinito

ï¿œltima tentativa: f(x)=x+e^{-|x|]
2011/2/11 Julio Cesar mailto:jcconegun...@gmail.com>>
f(x)=x-e^{-|x|} acho que agora vai.

2011/2/11 Julio Cesar mailto:jcconegun...@gmail.com>>
humm... tambï¿œm nï¿œo.


2011/2/11 Julio Cesar mailto:jcconegun...@gmail.com>>
ops... f(x)=x/2-1
2011/2/11 Julio Cesar mailto:jcconegun...@gmail.com>>

f(x)=x/2
2011/2/11 Jefferson Chan 
mailto:jeffersonj...@gmail.com>>

Alguem consegue pensar num exemplo de uma fun??o f:R-->R de classe
C^infinito tal que |f'(x)|<1  e f(x)!=x para todo x real?

abs,
Jefferson

=
Instru?es para entrar na lista, sair da lista e usar a lista em
http://www.mat.puc-rio.br/~obmlistas/obm-l.html
=



--
Julio Cesar Conegundes da Silva



--
Julio Cesar Conegundes da Silva



--
Julio Cesar Conegundes da Silva



--
Julio Cesar Conegundes da Silva



--
Julio Cesar Conegundes da Silva


[obm-l] RE: [obm-l] Re: [obm-l] RES: [obm-l] RE: [obm-l] RES: [obm-l] Equação de sétimo grau

2011-02-05 Por tôpico LEANDRO L RECOVA
A exponencial complexa deixa a prova mais compacta e elegante. Tambem pode-se 
usar o desenvolvimento de Taylor. Leandro Los Angeles, California. Sent from my 
HTC Touch Pro2 on the Now Network from Sprint®.


-Original Message-
From: Tiago
Sent: 2/5/2011 2:31:56 AM
To: obm-l@mat.puc-rio.br
Subject: [obm-l] Re: [obm-l] RES: [obm-l] RE: [obm-l] RES: [obm-l] Equação de 
sétimo grau
Não precisa usar exponencial complexa. A fórmula decorre das propriedades de 
seno e cosseno. Tente mostrar isso:

 (cos a + i sen a)(cos b + i sen b) = cos(a+b) + i sen(a+b)

A fórmula segue daí.

2011/2/4 Albert Bouskela mailto:bousk...@msn.com>>
Olá!

A Fórmula de De Moivre é decorrente da Fórmula de Euler:

e^(ix) = cis(x)
Lado esquerdo = Lado direito

Fazendo:  x = A/n

Lado esquerdo:  e^(iA/n) = (e^(iA))^(1/n)
Sabe-se que:  e^(iA) = cis(A)  ...  Fórmula de Euler
Logo:  (e^(iA))^(1/n) = (cis(A))^(1/n)

Lado direito:  cis(A/n)

Logo:  (cis(A))^(1/n) = cis(A/n)

Albert Bouskela
bousk...@msn.com<mailto:bousk...@msn.com>

De: owner-ob...@mat.puc-rio.br<mailto:owner-ob...@mat.puc-rio.br> 
[mailto:owner-ob...@mat.puc-rio.br<mailto:owner-ob...@mat.puc-rio.br>] Em nome 
de João Maldonado
Enviada em: 4 de fevereiro de 2011 21:15

Para: obm-l@mat.puc-rio.br<mailto:obm-l@mat.puc-rio.br>
Assunto: [obm-l] RE: [obm-l] RES: [obm-l] Equação de sétimo grau

 Peimeirament, obrigado pela solução =D

Nunca tinha ouvido falar dessa fórmula de De Moivre, achei muito interessante

cis(A)^n = cis(n.A), Há algum jeito fácil de provar isso?

[]'s
João






From: bousk...@msn.com<mailto:bousk...@msn.com>
To: obm-l@mat.puc-rio.br<mailto:obm-l@mat.puc-rio.br>
Subject: [obm-l] RES: [obm-l] Equação de sétimo grau
Date: Thu, 3 Feb 2011 20:23:53 -0200
Escrevendo de forma mais elegante:

Olá!

Você deve usar a Fórmula de De Moivre:

[ r (cos(A) + i sin(A) ]^(1/n) = r^(1/n) [ cos((A+2kpi)/n) + i sin((A+2kpi)/n) 
] , k=0, 1 ... (n-1)
   [ r (cis(A)) ]^(1/n) = r^(1/n) [ cis((A+2kpi)/n) ] , k=0, 1 ... (n-1)

Então:

x = 1^(1/7)

Escrevendo 1 na forma polar: 1 = 1 [ cos(0) + i sin(0) ]
   1 = 1 cis(0)

Logo: 1^(1/7) = 1^(1/7) [ cos((0+2kpi)/7) + i sin((0+2kpi)/7) ] , k=0, 1 ... 6
   1^(1/7) = 1^(1/7) [ cis((0+2kpi)/7) ] , k=0, 1 ... 6

Simplificando: 1^(1/7) = cos(2kpi/7) + i sin(2kpi/7) , k=0, 1 ... 6
   1^(1/7) = cis(2kpi/7) , k=0, 1 ... 6

Daí: x = { 1, cos(2pi/7) + i sin(2pi/7), cos(4pi/7) + i sin(4pi/7), cos(6pi/7) 
+ i sin(6pi/7), cos(8pi/7) + i sin(8pi/7), cos(10pi/7) + i sin(10pi/7), 
cos(12pi/7) + i sin(12pi/7) }

Albert Bouskela
bousk...@msn.com<mailto:bousk...@msn.com>

De: owner-ob...@mat.puc-rio.br<mailto:owner-ob...@mat.puc-rio.br> 
[mailto:owner-ob...@mat.puc-rio.br<mailto:owner-ob...@mat.puc-rio.br>] Em nome 
de João Maldonado
Enviada em: 3 de fevereiro de 2011 19:00
Para: obm-l@mat.puc-rio.br<mailto:obm-l@mat.puc-rio.br>
Assunto: [obm-l] Equação de sétimo grau

Há algum  jeito de resolver a equação de sétimo grau x^7 = 1 dentro dos 
complexos?

[]'s
João



--
Tiago J. Fonseca
http://legauss.blogspot.com


RE: [obm-l] Cônicas

2010-11-09 Por tôpico LEANDRO L RECOVA
Transforme de volta para coordenadas cartesianas. Sent from my HTC Touch Pro2 
on the Now Network from Sprint®.


-Original Message-
From: warley ferreira
Sent: 11/8/2010 8:07:42 PM
To: Lista de Discussão
Subject: [obm-l] Cônicas
Como faço para provar que a equação polar de uma cônica dada por 1/r = 1/h (1+E 
cos @)
determina uma hipérbole, parabola e elipse quando E >1, E = 1 e E < 1, 
respectivamente?
Desde já agradeço,
Abraços
Warley Souza




RE: [obm-l] Metodos Numericos

2010-10-08 Por tôpico LEANDRO L RECOVA
Numerical Analysis - Burden. Sent from my HTC Touch Pro2 on the Now Network 
from Sprint®.


-Original Message-
From: Adalberto Dornelles
Sent: 10/8/2010 7:58:11 PM
To: obm-l@mat.puc-rio.br
Subject: Re: [obm-l] Metodos Numericos
Olá Jeferson,

> Alguem poderia me indicar um livro de Metodos numericos mais aprofundado
> voltado para Computação alem do classico  -Cálculo Numérico  das autoras
> vera lucia e Marcia A. Gomes de preferencia em ingles desde ja agradeço!!

Tente esse:
Numerical recipes:
http://www.nr.com/

Abraço,
Adalberto

=
Instruções para entrar na lista, sair da lista e usar a lista em
http://www.mat.puc-rio.br/~obmlistas/obm-l.html
=


RE: [obm-l] Axioma ou teorema?

2010-09-27 Por tôpico LEANDRO L RECOVA

Outra forma de se ver isso esta no livro do Manfredo (Differential Geometry of 
Curves and Surfaces). Esse resultado e provado na secao de geodesicas. 
 

 


Date: Mon, 27 Sep 2010 04:10:21 -0700
From: luizfelipec...@yahoo.com.br
Subject: Re: [obm-l] Axioma ou teorema?
To: obm-l@mat.puc-rio.br






Olá,
 
Acho que, por envoltórias, vc prova em GE que o menor caminho é a reta. 
 
Q no fundo, é o que vc falou.
 
Abs
Felipe

--- Em sáb, 25/9/10, Tiago  escreveu:


De: Tiago 
Assunto: Re: [obm-l] Axioma ou teorema?
Para: obm-l@mat.puc-rio.br
Data: Sábado, 25 de Setembro de 2010, 17:25


Se estiver falando de geometria euclidiana, não sei qual seria a definição de 
caminho. Se caminho significar uma poligonal ligando A e B imagino que isto 
siga da desilgualdade triangular, que até onde me lembro é provada sem usar 
este fato.

Posso estar bastante enganado. ;-)


2010/9/25 Guilherme Vieira 


Caros colegas,

A afirmação "O menor caminho entre dois pontos A e B é o segmento de reta AB" é 
um axioma? Ou é um teorema?
Bem... creio que seja um axioma, pois me parece que não há como demonstrar o 
teorema, sem incorrer em petição de princípio.

Abraços!
Guilherme


-- 
Tiago J. Fonseca
http://legauss.blogspot.com

  

RE: [obm-l] Ajuda!!!

2010-09-16 Por tôpico LEANDRO L RECOVA
Warley, Calcule o numero total de maneiras e subtraia as possibilidades das 
criancas receberem 1,2,3, e 4 moedas. Acho que vai funcionar. Leandro Sent from 
my HTC Touch Pro2 on the Now Network from Sprint®.


-Original Message-
From: warley ferreira
Sent: 9/17/2010 12:43:28 AM
To: Lista de Discussão
Subject: [obm-l] Ajuda!!!
De quantas maneiras você pode distribuir 
[http://forum.obmep.org.br/latexrender/pictures/7b8b965ad4bca0e41ab51de7b31363a1.gif]
  moedinhas a 
[http://forum.obmep.org.br/latexrender/pictures/8ce4b16b22b58894aa86c421e8759df3.gif]
  crianças, se supõe-se que cada criança ganhe pelo menos 5?
Alguém poderia ajudar nesta questão!
Desde já agradeço!
Warley  Souza




RE: [obm-l] Ajuda Urgente!!!

2010-08-10 Por tôpico LEANDRO L RECOVA
Decomposicao em fracoes parciais. Sent from my HTC Touch Pro2 on the Now 
Network from Sprint®.


-Original Message-
From: warley ferreira
Sent: 8/10/2010 3:04:04 AM
To: Lista de Discussão
Subject: [obm-l] Ajuda Urgente!!!
Oá Pessoal, td bom?
Como calcular a soma abaixo?
1/1.2 + 1/2.3 + ... + 1/(n-1).n
Desde já agradeço,
Abraços
Warley F Souza




[obm-l] Re: [obm-l] Вам необходимо знать все о в ашей фамилии

2010-05-25 Por tôpico Leandro Lima
Russo? 
Como nao sei russo, usei o tradutor do google. Ficou assim:
Talvez o seu nome e mantido em segredo, que seus avos nao queriam falar!


   Login

To: nico...@mat.puc-rio.br 
Subject: [obm-l] Вам необходимо знать все о вашей фамилии


Возможно, ваша фамилия хранит тайны, которые ваши бабушки и дедушки не хотели 
рассказывать! 

Возможно, ваша фамилия хранит тайны, которые ваши бабушки и дедушки не хотели 
рассказывать! 

Вход на сайт


[obm-l] RE: [obm-l] Derivaçã o implícita

2010-05-04 Por tôpico LEANDRO L RECOVA

Rodrigo,

 

Acho que muitas das aplicacoes vem mais tarde num curso de analise, geometria 
diferencial, topologia diferencial, sistemas dinamicos e outros assuntos. O 
Elon esta dando uma aula de funcoes implicitas e os videos estao no site do 
IMPA. 

 

Nao sei que curso voce esta fazendo este momento, mas num curso de analise no 
R^n fica tudo claro. Eu tambem ficava ancioso em ver aplicacoes, e quando vi as 
primeiras aplicacoes foi num curso de geometria diferencial.

 

Leandro.

 
> Date: Tue, 4 May 2010 16:43:54 -0400
> Subject: [obm-l] Derivação implícita
> From: rossoas...@gmail.com
> To: obm-l@mat.puc-rio.br
> 
> Amigos, a dúvida é conceitual mesmo. Não tenho tido mta dificuldade
> pra resolver os problemas, mas a dúvida me persegue sem que o
> professor tenha me feito entender. Afinal de contas qual a lógica
> (implícita ou não) da derivação implícita? Quando usá-la? Porque
> usá-la?
> 
> grato,
> Rodrigo
> 
> =
> Instruções para entrar na lista, sair da lista e usar a lista em
> http://www.mat.puc-rio.br/~obmlistas/obm-l.html
> =
  

[obm-l] Conjuntos enumeráveis

2010-04-26 Por tôpico Leandro Lima

Caros,

peço ajuda para resolver os seguintes exercícios do livro Curso de Análise - 
Vol. 1 - do prof. Elon L. Lima:

1)

a)  Se X é finito e Y é enumerável, então F(X;Y) é enumerável. (F(X;Y) é o 
conjunto de todas as funções f: X -> Y.)

b) Para cada função f : N -> N seja A_f = {n pertencente a N; f(n) diferente de 
1}.

Prove que o conjunto X das funções f : N -> N tais que A_f é finito é um 
conjunto enumerável.


2) Prove que o conjunto das sequências crescentes (n1 < n2 < n3 < ...) de 
números naturais não é enumerável.


 
Desde já, 

grato!
 
  

     Leandro Lima. 




[obm-l] Re: [obm-l] Números Primos

2010-04-09 Por tôpico Leandro Lima
Olá, Vitor!

A média aritmética de dois números primos pode ser um número primo!

Por exemplo: 
Dado a primo, (a + a)/2  = a; 
Ou, (7 + 3)/2 = 5;
Ou, (101 + 5)/2 = 53.

Mas, também pode a média aritmética entre dois primos não ser um primo.
Por exemplo: 

(5 + 7)/2 = 6;
Ou, (1001 + 3) = 52. 


  Abraço!
 Leandro. 





From: vitor alves 
Sent: Friday, April 09, 2010 8:00 AM
To: obm-l@mat.puc-rio.br 
Subject: [obm-l] Números Primos


Como provar que a média aritmética de dois números primos nunca é um número 
primo? 


Quer ver seus e-mails de todas as contas num lugar só? Junte todas elas no 
Hotmail. 

RE: [obm-l] EDO

2010-04-01 Por tôpico LEANDRO L RECOVA

Eu nao tinha lapis e papel aqui do lado. Fiz direto no computador.

 

Fique a vontade para fazer qualquer correcao. A ideia principal esta no email 
anterior.

 

Obrigado por detectar o erro no delta. Os casos particulares podem ser 
derivados a partir dai. 

 

Leandro.
 
> Date: Thu, 1 Apr 2010 14:31:10 -0300
> Subject: Re: [obm-l] EDO
> From: mffmartine...@gmail.com
> To: obm-l@mat.puc-rio.br
> 
> Creio que você errou no delta da equação do segundo grau.
> 
> Teríamos:
> 
> x1 = [1-sqrt(- 4h+1)]/2.
> x2 = [1+sqrt(- 4h+1)]/2.
> 
> E aqui devemos considerar também o caso em que delta é negativo (h < 1/4)!
> 
> =
> Instruções para entrar na lista, sair da lista e usar a lista em
> http://www.mat.puc-rio.br/~obmlistas/obm-l.html
> =
  

RE: [obm-l] EDO

2010-04-01 Por tôpico LEANDRO L RECOVA

Wagner, se h >0, entao podemos fazer o seguinte, 

 

x(1-x)-h = x - x^2 - h = -(x-x1)(x-x2)

 

onde

 

x1 = 1-sqrt(4h+1)/2.

x2 = 1+sqrt(4h+1)/2.

 

Trabalhe com x1 e x2 durante todo o tempo agora e no final substitua na 
equacao. Entao,

 

x' = -(x-x1)(x-x2). 

 

x'/[(x-x1)(x-x2)] = -1.  (1)

 

Agora, tente usar fracoes parciais no lado esquerdo.

 

1/(x-x1)(x-x2) =  A/x-x1  +  B/x-x2.

 

(A+B)x = 0  => A=-B (ja que x e diferente de zero).

-Ax2 - Bx1 = 1 => B(x2-x1)=1 => B=1/(x2-x1). 

 

Portanto, use A e B no que segue e substitua no final. A equacao (1) fica

 

Ax'/x-x1 + Bx'/x-x2 = -1  (Integre ambos os lados em relacao a t, supondo t e 
sua variavel independente),

 

A log(x-x1) + B log(x-x2) = -t + C (Use o fato de que A=-B) onde C e uma 
constante de integracao.

A log(x-x1) - A log(x-x2) = -t + C

 

log[(x-x1)/(x-x2)] = -t/A + C/A

 

(x-x1)/(x-x2) = exp(-t/A). K,   where K=exp(C/A). You find K based on the 
initial conditions of the problem.

 

Agora, eu deixo como exercicio voce isolar o x. 

 

Leandro.

Los Angeles, California.

 

 

 

 

 

 

 

 

 

 

 

 

 

 

 

 

 

 


 


Date: Wed, 31 Mar 2010 13:00:56 -0700
From: lulu...@yahoo.com.br
Subject: [obm-l] EDO
To: obm-l@mat.puc-rio.br; fcostabarr...@gmail.com; ralp...@gmail.com






Olá Pessoal, td bom?
Queria uma ajuda para resolver a Eq. Diferencial Logística x' = x (1-x) - h, 
onde h é parámetro positivo.
Desde já agradeço
abraços
 Wagner Luis Souza


Veja quais são os assuntos do momento no Yahoo! + Buscados: Top 10 - 
Celebridades - Música - Esportes 

RE: [obm-l] Teorema sobre "rank" de matrizes

2010-03-30 Por tôpico LEANDRO L RECOVA

Lucas,

 

"Rank" quer dizer o posto da matriz mxn. Basicamente, se voce tem uma 
transformacao linear T de um espaco em T:R^m -> R^{n} , o posto vai te dizer 
qual e a dimensao da imagem dessa transformacao. Como cada coluna da matriz 
associada a T e a imagem de um dos vetores da base canonica em R^{m}, entao, o 
numero de colunas linearmente independentes vai lhe fornecer informacao sobre a 
dimensao da imagem.  O livro do Elon tem uma excelente explicacao sobre o 
assunto e inclusive prova o que voce esta querendo. 

 

Leandro.
 


From: luca...@dcc.ufba.br
Date: Tue, 30 Mar 2010 10:51:16 -0300
Subject: [obm-l] Teorema sobre "rank" de matrizes
To: obm-l@mat.puc-rio.br

Olá,

eu estava resolvendo os exercícios do livro "Introdução a algoritmos" de Cormen 
et al. E encontrei o que eu acredito ser um erro.

No livro, a definição dita alternativa para o "rank" (não sei traduzir) de uma 
matriz 'A' mxn é o maior valor 'r' tal que existam duas matrizes (uma mxr e 
outra rxn) tais que seu produto seja igual à 'A'. (A definição principal é a de 
que uma matriz tem rank 'r' se existirem no máximo 'r' linhas/colunas 
linearmente independentes).

O livro também fala que para uma matriz 'A' mxn, rank(A) <= min(m, n).

Então, na questão 31.1-9, é pedido pra provar que rank(AB) <= min( rank(A), 
rank(B) ).
No entanto, eu consegui provar que min( rank(A), rank(B) ) <= rank(AB)

Este é meu argumento:
Seja 'A' uma matriz mxk
e seja 'B' uma matriz kxn

Então rank(AB) >= k, já que é possível multiplicar duas matrizes mxk e kxn pra 
encontrar AB, mas não se sabe se é possível encontrar duas matrizes de maiores 
dimensões para se obter AB. (Ver definição acima)

Sabemos que rank(A) <= min(m, k) e que rank(B) <= min(k, n)
E sabemos que k >= min(m, k) e que k >= min(k, n)

Para a matriz A, temos:
rank(A) <= min(m, k) <= k <= rank(AB)

Portanto, rank(A) <= rank(AB). De forma análoga, rank(B) <= rank(AB) e, 
portanto,
rank(AB) >= max( rank(A), rank(B) ) >= min( rank(A), rank(B) )


Eu cometi algum engano? Se eu realmente cometi e alguém pudesse responder este 
exercício pra mim, eu ficaria grato ;)
  

[obm-l] RE: [obm-l] Questão do IME

2010-03-15 Por tôpico LEANDRO L RECOVA

Marcone,

 

O enunciado esta correto? 5x^2+2x^2 no primeiro membro? Seria 7x^2? Podes 
confirmar?
 


From: marconeborge...@hotmail.com
To: obm-l@mat.puc-rio.br
Subject: [obm-l] Questão do IME
Date: Tue, 16 Mar 2010 01:20:14 +



O par ordenado (x,y),com x e y inteiros positivos,satisfaz a equação 
5x^2+2x^2=11*(xy-11).O valor de x+y é
a)160  b)122 c)81  d)41 e)11
Considerei q x e y n podem ser ambos pares nem ambos ímpares.E q x é ímpar e y 
é par,pois se x fosse par e y,ímpar teriamos o primeiro membro par e o 
segundo,ímpar,um absurdo.Dai,veriquei q o primeiro membro é múltiplo de 4 mais 
1.Como o segundo membro é claramente múltiplo de 11,procurei um número nessas 
condições,para depois encontrar x e y.Mesmo após várias tentativas,n 
consegui...Há caminhos melhores.Alguem ajudaria?
 



Acesse todas as suas contas de e-mail num único login dentro do Hotmail. Veja 
como.   

[obm-l] RE: [obm-l] Re: [obm -l] Média Aritmética e Geométrica

2010-03-07 Por tôpico LEANDRO L RECOVA

Lembre que 

 

(x-y)^2 > 0. 

 

x^2-2xy+y^2 > 0

 

x^2 - 4xy + 2xy + y^2 > 0

 

Isola o termo 4xy,

 

4xy < (x+y)^2

 

E o resultado segue tirando a raiz quadrada em ambos os lados. 

 

Leandro
 


Date: Sat, 6 Mar 2010 22:16:22 -0300
Subject: [obm-l] Re: [obm-l] Média Aritmética e Geométrica
From: fcostabarr...@gmail.com
To: obm-l@mat.puc-rio.br

Para provar que x > y, você pode provar que  x - y > 0. Acho que assim dá certo.


Em 6 de março de 2010 16:01, Emanuel Valente  
escreveu:

Pessoal, eu tinha feito esse exercício no cursinho, mas não lembro por
onde saí. Alguma luz?

Sejam x,y numeros reais positivos. Prove que:

sqrt(x.y) < (x+y)/2

--
Emanuel

=
Instruções para entrar na lista, sair da lista e usar a lista em
http://www.mat.puc-rio.br/~obmlistas/obm-l.html
=

  

RE: [obm-l] P.A

2009-12-28 Por tôpico LEANDRO L RECOVA

Escolha os numeros da forma: 

 

x-2r,x-r,x,x+r,x+2r.

 

A soma deles e dada por: (x-2r)+(x-r)+x+(x+r)+(x+2r)=5 => 5x=5 =>x=1. 

 

Agora, determinar a razao r usando a segunda condicao:

 

[1/x-2r] + [1/x-r] + 1/x + [1/x+r]+ 1/[x+2r] = 563/63.

 

Substituta x=1, entao,

 

1/1-2r + 1/1-r + 1 + 1/1+r + 1/1+2r = 563/63

 

Agora fica facil.

 

Leandro.
 


Date: Sat, 26 Dec 2009 10:06:37 -0800
From: uizn...@yahoo.com.br
Subject: [obm-l] P.A
To: obm-l@mat.puc-rio.br





Questão 18 do livro P.A , P.G e matrizes de fundamentos de matemática 
elementar. IEZZI.
18)Obtenha 5 números reais em P.A, sabendo que sua soma é 5 e a soma de seus 
inversos  é 563/63.

Alguem pode fazer ou explicar uma forma fácil de resolver essa questão!! 
Obrgado!



Veja quais são os assuntos do momento no Yahoo! + Buscados: Top 10 - 
Celebridades - Música - Esportes 

RE: [obm-l] um limite

2009-05-07 Por tôpico LEANDRO L RECOVA

 

sin(a+b) = sin(a)cos(b)+cos(a)sin(b)

sin(a-b) = sin(a)cos(b)-cos(a)sin(b)

 

sin(a+b) - sin(a-b) = 2.cos(a).sin(b)

 

Faca

 

a+b = x^2 + 1/x e 

a-b = 1/x

 

Entao, 

 

2a = x^2 + 2/x

 

a = [x^2 + 2/x]/2

 

2b = x^2

b= x^2/2

 

Entao,

 

f(x)=[sin(x^2+1/x) - sin(1/x)]/x = 2cos(x^2/2 + 1/x).sin(x^2/2)/x 

 

Ja que x e diferente de zero, podemos multiplicar e dividir o numerador por x e 
obter

 

f(x)= x. cos(x^2/2 + 1/x).sin(x^2/2)/(x^2/2)

 

Observe que cos e uma funcao limitada, o segundo termo e o limite fundamental 
do seno que tende para 1. 

 

lim f(x) = 0.

x->0

 

Faca um grafico no Excel ou MATLAB.

 


 


Date: Thu, 7 May 2009 10:56:06 -0700
From: adt...@yahoo.com.br
Subject: [obm-l] um limite 
To: obm-l@mat.puc-rio.br





Alguém poderia me ajudar neste limite por favor:

LIMITE QUANDO X TENDE A ZERO DE F:

F(x)= [ sen(x^2+1/x) - sen (1/x)  ]/x

Desde já agradeço



--- Em qua, 6/5/09, Paulo Santa Rita  escreveu:

De: Paulo Santa Rita 
Assunto: Re: [obm-l] serie para ln(2)
Para: obm-l@mat.puc-rio.br
Data: Quarta-feira, 6 de Maio de 2009, 10:48

Ola Bernardo e demais colegas
desta lista ... OBM-L,

1 ) Muito boa a sua mensagem, mas note que nao foi isso que o Luis
Lopes perguntou e, portanto, nao foi sobre o que eu respondi. Pelo que
eu entendi, o Luis quer saber se a colocao arbitraria de colchetes vai
afetar o valor original da
 serie ( sua soma), vale dizer, NAO HA A
HIPOTESE DE MESMO NUMERO DE TERMOS e, pior ainda, OS SINAIS DOS TERMOS
SAO PRESERVADOS, ou seja, ele simplesmente introduz colchetes e outros
delimitadores, SEM ALTERAR OS SINAIS ORIGINAIS, o que afeta o
resultado ainda mais.

2 ) Nao, amigo, descupe. Devo ter me expressado mal. Quero dizer :

Dado 2P, construa o triangulo tal que :
Soma das medianas = 2P
perimetro = 2P

Eu nao pensei sobre a questao. Nem sei se e trivial ou trabalhosa. Ela
veio a minha cabeca quando respondia o Luis Lopes. Como o Luis e um
Mestre consumado no assunto, queria saber se tal questao ja constava
no livro dele ou se ele dispunha de tempo pra apresentar uma solucao,
caso exista.

Um Abracao
PSR, 40605091048



2009/5/6 Bernardo Freitas Paulo da Costa :
> Grande Paulo ! Mas eu gostaria de tentar dar um
"palpitezinho"... e
>
 vou acabar dando dois :
>
> 2009/5/6 Paulo Santa Rita :
>> Ola Luis e demais colegas
>> desta lista ... OBM-L,
>>
>> 1) O valor de uma serie ( sua "soma" ) e formalmente
definido como em
>> qualquer serie, vale dizer, como limite das suas reduzidas. Assim, se
>> :
>>
>> S = a1 + a2 + a3 + ... Â = L
>>
>> Entao a sequencia Rn = a1 + ... + an e tal que Lim Rn = L. Neste
>> particular caso que voce cita o agrupamento nao vai causar problemas ,
>> conforme pode-se ver considerando as suas reduzidas. No caso geral,
>> PRESERVANDO-SE O SINAL DOS TERMOS, nao pode. Basta , mais uma vez,
>> considerar as reduzidas.
>>
>> A resposta mais geral seria, entao, assim:
>>
>> "Se e possivel provar que a sequencia das reduzidas
 original
converge
>> para o mesmo valor que a nova sequencia das reduzidas ( aquelas
>> derivadas do agrupamento ) entao o agrupamento e valido"
>>
>> 2) Estas series sao melhor tratadas em espacos de sobolev
> Eu tenho muito pouca prática com Sobolev, mas se você pudesse
> detalhar... Eu vou detalhar o que eu pensei quando escrevi :
>
> Temos uma soma S, com termos a1, a2, a3, ... Faça a seguinte
hipótese
> : a_n tende a zero. Então, agrupando os termos SEMPRE EM MESMO NUMERO,
> se a primeira soma existir, a agrupada também, e vale o mesmo valor. A
> idéia, é exatamente voltar à definição que o Paulo deu mais
acima, e
> notar que um grupo equivale a somar N termos sucessivos. Logo, somar n
> grupos, equivale a tomar a nN-ésima parcial. Sabemos que a
seqüência
> das parciais converge, logo a das
 "N"-parciais também, e para
o mesmo
> valor, pois é uma subseqüência. Muito mais interessante é
tentar a
> recíproca : se a soma dos agrupamentos converge, o que acontece com a
> soma original ? E é aí que a hipótese inicial tem todo o seu
sentido:
> sabemos que a subseqüência S_{Nk} converge para S, e queremos
estudar
> a seqüência completa S_n. Ora, como os termos tendem a zero, de
S_{Nk
> + 1} = S_{Nk} + a_{Nk + 1}, tomando o limite no termo da direita (que
> existe !), vemos que lim S_{Nk + 1} existe também, e vale S + lim
> a_{Nk + 1} = S. Como, além disso, temos um número finito de restos
> modulo N, e todas as subseqüências S_n{Nk + a} convergem para S
(faça
> uma depois da outra !!), a seqüência original também converge
para S,
> pois a partir de um certo N_0, todos os S_{Nk} estão a epsilon
 de S,
> para um N_1, os S_{Nk + 1}, etc e tal, logo para o máximo dos N_a,
> todo mundo está a epsilon de S. Note que a finitude (que vem da
> regularidade dos parênteses) é crucial, bem como a hipótese que
os
> termos tendem a zero !!
>
>> 3) No fim da minha mensagem eu citei uma solucao diferente da que
>> apresentei, usando polinomios ( que, inclusive, gneraliza a questao

[obm-l] RE: [obm-l] Prova do teorema fundamenta da álgebra por análi se complexa

2009-04-04 Por tôpico LEANDRO L RECOVA

E uma belissima prova e usa o teorema de Liouville. Voce precisa estudar um 
pouco sobre a Integral de Cauchy. O livro do Churchill de Variaveis Complexas 
tem a demonstracao. 

 

Leandro.
 
> Date: Thu, 26 Mar 2009 17:37:25 -0700
> From: ana...@yahoo.com
> Subject: [obm-l] Prova do teorema fundamenta da álgebra por análise complexa
> To: obm-l@mat.puc-rio.br
> 
> 
> Oi a todos
> 
> Soube que, por análise complexa, há uma prova do teorema fundamental da 
> álgebra bem mais curta do que por álgebra. Alguém a conhece? Exige 
> conhecimentos profundos?
> 
> Obrigada
> Ana
> 
> 
> 
> 
> 
> 
> 
> 
> 
> 
> 
> 
> =
> Instruções para entrar na lista, sair da lista e usar a lista em
> http://www.mat.puc-rio.br/~obmlistas/obm-l.html
> =


RE: [obm-l] Wavelet

2009-03-30 Por tôpico LEANDRO L RECOVA

Na pagina do Gilbert Strang tem algo interessante:

 

http://www-math.mit.edu/~gs/papers/papers.html 

 

Outros artigos que voce pode procurar sao os da Ingrid Daubechies e do 
Professor Mallat. 

 

Leandro.
 


Date: Mon, 30 Mar 2009 05:17:01 -0700
From: regisgbar...@yahoo.com.br
Subject: [obm-l] Wavelet
To: obm-l@mat.puc-rio.br





Olá Pessoal

Gostaria de ebooks e sites sobre wavelet pois tenho interesse em conhecer esta 
área.

atenciosamente

regis

P.S.
Mostrar que: (todas as grandezas são vetores)
[(A x P).(B x Q)x(C x R)] + [(A x Q).(B x R)x(C x P)] + [(A x R).(B x P)x(C x 
Q)] = 0
 
onde o símbolo 'x' indica produto vetorial e o símbolo '.' indica produto 
escalar.


O problema a seguir foi publicada nessa lista assim gostaria de saber o email 
do autor desse problema.




Veja quais são os assuntos do momento no Yahoo! + Buscados: Top 10 - 
Celebridades - Música - Esportes

RE: [obm-l] questões topologia da reta

2009-01-27 Por tôpico LEANDRO L RECOVA

Eu esqueci de escrever que X = UNIAO_{1 a n} I_{xi} intersecao X. Desculpe. 

From: leandrorec...@msn.comto: ob...@mat.puc-rio.brsubject: RE: [obm-l] 
questões topologia da retaDate: Mon, 26 Jan 2009 13:36:41 -0800

Primeiro exercicio: Ja que X e compacto, voce consegue uma cobertura finita de 
intervalos I_{xi} com centro em x_{i} tal que X esta na uniao desses 
intervalos. Voce tambem pode escrever X = Intersecao de I_{xi} com X. Agora, 
como f e localmente limitada, entao ela e limitada em cada f(I_{xi} intersecao 
X). Deixo a conclusao pra voce. Regards, Leandro 

Date: Sun, 25 Jan 2009 21:16:57 -0200Subject: [obm-l] questões topologia da 
retaFrom: murilo.kr...@gmail.comto: ob...@mat.puc-rio.brprezados,estou 
apanhando nessas duas questões, alguém poderia me dar uma força? Seja X C R. 
Uma funcão f : X -> R chama-se locamente limitada quando para cadax 
pertencente a X existe um intervalo aberto Ix, contendo x, talque f I Ix 
(interseção) X e limitada. Mostre quese X é compacto, toda função f : X -> R 
localmente limitada e limitada. Prove que a soma da serie cujos termos são os 
comprimentos dos intervalos omitidos paraformar o conjunto de Cantor é igual a 
1.abraços,Murilo

RE: [obm-l] questões topologia da reta

2009-01-26 Por tôpico LEANDRO L RECOVA

Primeiro exercicio:
 
Ja que X e compacto, voce consegue uma cobertura finita de intervalos I_{xi} 
com centro em x_{i} tal que X esta na uniao desses intervalos. Voce tambem pode 
escrever X = Intersecao de I_{xi} com X. Agora, como f e localmente limitada, 
entao ela e limitada em cada f(I_{xi} intersecao X). Deixo a conclusao pra 
voce.
 
Regards,
 
Leandro
 



Date: Sun, 25 Jan 2009 21:16:57 -0200Subject: [obm-l] questões topologia da 
retaFrom: murilo.kr...@gmail.comto: ob...@mat.puc-rio.brprezados,estou 
apanhando nessas duas questões, alguém poderia me dar uma força? Seja X C R. 
Uma funcão f : X -> R chama-se locamente limitada quando para cadax 
pertencente a X existe um intervalo aberto Ix, contendo x, talque f I Ix 
(interseção) X e limitada. Mostre quese X é compacto, toda função f : X -> R 
localmente limitada e limitada. Prove que a soma da serie cujos termos são os 
comprimentos dos intervalos omitidos paraformar o conjunto de Cantor é igual a 
1.abraços,Murilo

RE: [obm-l] [OFF] perseguicao

2009-01-22 Por tôpico LEANDRO L RECOVA

Isso e uma ofensa ao Professor Terence Tao aqui de UCLA.
 
Nao respondam essas mensagens. 
 
Regards,
 
Leandro.



Date: Thu, 22 Jan 2009 22:21:06 -0300Subject: Re: [obm-l] [OFF] 
perseguicaoFrom: fgam...@gmail.comto: ob...@mat.puc-rio.bro primeiro colocado 
já está morto?
2009/1/22 Felipe Diniz 
Respeitem o "maior especialista do mundo em Formulas para Numeros Primos."



On Thu, Jan 22, 2009 at 9:42 PM, João Maldonado  
wrote:

Cada uma...> Date: Thu, 22 Jan 2009 06:24:17 -0800> From: 
mathfire2...@yahoo.com.br> Subject: [obm-l] [OFF] perseguicao> To: 
obm-l@mat.puc-rio.br


> > Ola,> > Venho comunicar que apos eu obter a setima colocacao brasileira> na 
> > Olimpiada Iberoamericana de Matematica Universitaria em 2006> fui 
> > perseguido das mais diversas formas. Fui preso e drogado de> forma covarde, 
> > tendo serios danos a saude provacados por drogas> que me obrigaram a tomar. 
> > Hoje tenho dificuldade para manter a> atencao, para ler e para estudar. 
> > Venho tentando a meses denunciar> esta situacao junto as autoridades, mas, 
> > ao que parece elas sao> coniventes com o que esta ocorrendo.> > Peco a 
> > ajuda de algum membro da lista.> > A situacao esta insustentavel e suspeito 
> > que tentarao me matar,> fazendo parecer um problema de saude meu ou um 
> > acidente, ou> ainda fazendo parecer que o responsavel por minha morte seja 
> > eu > mesmo.> > Abracos.> > > Eric Campos> 
> > => DEUS=MATEMATICA> Eric Campos 
> > Bastos Guedes - O maior> especialista do mundo em Formulas para > Numeros 
> > Primos.> Endereco: RUA DOMINGUES DE SA, 422> ICARAI - NITEROI - RJ - CEP: 
> > 24220-091> BRAZIL> mathf...@gmail.com> mathfire2...@yahoo.com.br> MSN: 
> > fato...@hotmail.com> => > > > 
> > Veja quais são os assuntos do momento no Yahoo! +Buscados> 
> > http://br.maisbuscados.yahoo.com> > 
> > => 
> > Instruções para entrar na lista, sair da lista e usar a lista em> 
> > http://www.mat.puc-rio.br/~obmlistas/obm-l.html> 
> > =

É fácil compartilhar suas fotos com o Windows LiveT Arraste e solte

RE: [obm-l] Discussão Equação 3o. Graus

2009-01-12 Por tôpico LEANDRO L RECOVA

Tente trabalhar com as relacoes de Girard! 

Date: Mon, 12 Jan 2009 07:21:05 -0800From: luizfelipec...@yahoo.com.brsubject: 
[obm-l] Discussão Equação 3o. GrausTo: obm-l@mat.puc-rio.br




Pessoal,
 
Qual a condição para que a equação abaixo tenha raízes inteiras positivas ? Dá 
para determinar a forma geral da solução desta equação ?
 
Z3 – aZ - 3b K= 0


Veja quais são os assuntos do momento no Yahoo! + Buscados: Top 10 - 
Celebridades - Música - Esportes

RE: [obm-l] Continuação Analítica

2009-01-06 Por tôpico LEANDRO L RECOVA

Henrique,
 
O Stein escreveu um livro mais recente sobre Analise Complexa. Este deve ter o 
que voce esta procurando: 
http://www.amazon.com/Complex-Analysis-Princeton-Lectures/dp/0691113858/ref=pd_sim_b_5.
 Alias, o Stein escreve muito bem e de forma elegante. Eu gostei da forma como 
ele aborda, por exemplo, a demonstracao do Teorema da Diferenciacao de 
Lebesgue. De uma olhada no livro Singular Integrals dele se voce tiver 
interesse.
 
Regards,



Date: Tue, 6 Jan 2009 12:28:09 -0200From: henrique.re...@gmail.comto: 
ob...@mat.puc-rio.brsubject: Re: [obm-l] Continuação AnalíticaObrigado pelos 
links, mas acho que são os mesmos. Qual seria a obra do Stein?
2009/1/5 LEANDRO L RECOVA 

Alfhors: 
http://www.amazon.com/Complex-Analysis-Lars-Ahlfors/dp/0070006571/ref=pd_bbs_sr_1?ie=UTF8&s=books&qid=1231173649&sr=8-1
   Stein: 
http://www.amazon.com/Complex-Analysis-Lars-Ahlfors/dp/0070006571/ref=pd_bbs_sr_1?ie=UTF8&s=books&qid=1231173649&sr=8-1
   

Date: Mon, 5 Jan 2009 11:12:37 -0200
From: henrique.re...@gmail.comto: ob...@mat.puc-rio.brsubject: Re: [obm-l] 
Continuação Analítica


Você poderia passar os nomes completos dos autores e os nomes das obras para 
que eu possa procurar? Obrigado pelas indicações.
2009/1/4 LEANDRO L RECOVA 

O livro do Alfhors ou do Stein tem bastante material.

Date: Sun, 4 Jan 2009 10:46:16 -0200From: henrique.re...@gmail.comto: 
ob...@mat.puc-rio.brsubject: [obm-l] Continuação Analítica 
Alguém teria indicações de livros sobre Continuação Analítica ou que contenham 
partes dedicadas a esse assunto?-- Henrique-- Henrique-- Henrique

RE: [obm-l] Continuação Analítica

2009-01-06 Por tôpico LEANDRO L RECOVA

Desculpa Henrique. O livro se chama Harmonic Analysis 
http://www.amazon.com/Harmonic-Analysis-Elias-M-Stein/dp/0691032165/ref=pd_bbs_sr_1?ie=UTF8&s=books&qid=1231277899&sr=8-1.
 
 
Ele e um professor de Princeton e o Nicolau deve conhece-lo. Eu estudei outra 
obra dele sobre Singular Integrals, mas somente o primeiro capitulo. 
 
Regards,
 
Leandro.



Date: Tue, 6 Jan 2009 12:28:09 -0200From: henrique.re...@gmail.comto: 
ob...@mat.puc-rio.brsubject: Re: [obm-l] Continuação AnalíticaObrigado pelos 
links, mas acho que são os mesmos. Qual seria a obra do Stein?
2009/1/5 LEANDRO L RECOVA 

Alfhors: 
http://www.amazon.com/Complex-Analysis-Lars-Ahlfors/dp/0070006571/ref=pd_bbs_sr_1?ie=UTF8&s=books&qid=1231173649&sr=8-1
   Stein: 
http://www.amazon.com/Complex-Analysis-Lars-Ahlfors/dp/0070006571/ref=pd_bbs_sr_1?ie=UTF8&s=books&qid=1231173649&sr=8-1
   

Date: Mon, 5 Jan 2009 11:12:37 -0200
From: henrique.re...@gmail.comto: ob...@mat.puc-rio.brsubject: Re: [obm-l] 
Continuação Analítica


Você poderia passar os nomes completos dos autores e os nomes das obras para 
que eu possa procurar? Obrigado pelas indicações.
2009/1/4 LEANDRO L RECOVA 

O livro do Alfhors ou do Stein tem bastante material.

Date: Sun, 4 Jan 2009 10:46:16 -0200From: henrique.re...@gmail.comto: 
ob...@mat.puc-rio.brsubject: [obm-l] Continuação Analítica 
Alguém teria indicações de livros sobre Continuação Analítica ou que contenham 
partes dedicadas a esse assunto?-- Henrique-- Henrique-- Henrique

RE: [obm-l] Continuação Analítica

2009-01-05 Por tôpico LEANDRO L RECOVA

Alfhors: 
http://www.amazon.com/Complex-Analysis-Lars-Ahlfors/dp/0070006571/ref=pd_bbs_sr_1?ie=UTF8&s=books&qid=1231173649&sr=8-1
  
 
Stein: 
http://www.amazon.com/Complex-Analysis-Lars-Ahlfors/dp/0070006571/ref=pd_bbs_sr_1?ie=UTF8&s=books&qid=1231173649&sr=8-1
 
 
 



Date: Mon, 5 Jan 2009 11:12:37 -0200From: henrique.re...@gmail.comto: 
ob...@mat.puc-rio.brsubject: Re: [obm-l] Continuação AnalíticaVocê poderia 
passar os nomes completos dos autores e os nomes das obras para que eu possa 
procurar? Obrigado pelas indicações.
2009/1/4 LEANDRO L RECOVA 

O livro do Alfhors ou do Stein tem bastante material.

Date: Sun, 4 Jan 2009 10:46:16 -0200From: henrique.re...@gmail.comto: 
ob...@mat.puc-rio.brsubject: [obm-l] Continuação Analítica
Alguém teria indicações de livros sobre Continuação Analítica ou que contenham 
partes dedicadas a esse assunto?-- Henrique-- Henrique

RE: [obm-l] Continuação Analítica

2009-01-04 Por tôpico LEANDRO L RECOVA

O livro do Alfhors ou do Stein tem bastante material.

Date: Sun, 4 Jan 2009 10:46:16 -0200From: henrique.re...@gmail.comto: 
ob...@mat.puc-rio.brsubject: [obm-l] Continuação AnalíticaAlguém teria 
indicações de livros sobre Continuação Analítica ou que contenham partes 
dedicadas a esse assunto?-- Henrique

RE: [obm-l] 1+2+3+4+5+...

2008-12-22 Por tôpico LEANDRO L RECOVA

A primeira serie e a serie harmonica e ela e divergente. 
 
Tem um documento interessante a esse respeito: 
http://ferrari.dmat.fct.unl.pt/services/AnalMat2A/AMII-A-2004-TE-Cap3.pdf 
 
Voce tambem pode ver isso em qualquer livro de Calculo ou analise.
 
Regards,



From: markitov...@hotmail.comto: ob...@mat.puc-rio.brsubject: [obm-l] 
1+2+3+4+5+...Date: Mon, 22 Dec 2008 00:57:43 +

Olá, pessoal!A um bom tempo atrás, li sobre uma "prova" feita por Ramanujam de 
que 1+2+3+4+5+...=-1/12Recentemente fiquei interessado em olhar a prova xD, 
alguém a conheceria?Também fiquei interessando na "prova" de Euler de que 
1-2+3-4+5-6+...=1/4, alguém conheceria esta também?Muito obrigado pela atenção!



Instale a Barra de Ferramentas com Desktop Search e ganhe EMOTICONS para o 
Messenger! É GRÁTIS!

RE: [obm-l] Transformada de Fourier

2008-11-25 Por tôpico LEANDRO L RECOVA

Seria talvez interessante entao voce ler algo sobre Wavelets tambem. O livro do 
Burrus, ou do Strang, ou Daubechies sao boas referencias. O Stephanne Mallat 
tambem tem alguns artigos interessantes.

Date: Tue, 25 Nov 2008 14:07:24 -0200From: [EMAIL PROTECTED]: [EMAIL 
PROTECTED]: Re: [obm-l] Transformada de FourierRalph, estou lendo sobre a 
transformada de fourier exatamente por causa de processamento de imagens. Vou 
ler sua referencia,ObrigadoDenisson
2008/11/25 Ralph Teixeira <[EMAIL PROTECTED]>

Talvez nao seja exatamente o ideal, mas aqui tem alguma coisa que eu escrevi 
voltado para Processamento de Imagens:
 
http://www.visgraf.impa.br/Courses/eescala/index.html
 
A preocupacao era mais com aplicacoes do que com teoria, mas estah lah.
 
Abraco,
 Ralph



On Mon, Nov 24, 2008 at 11:03 PM, Denisson <[EMAIL PROTECTED]> wrote:
Alguém poderia citar referências sobre a transformada de fourier?. Serve em 
inglês, portugues ou espanhol.-- Denisson-- Denisson

RE: [obm-l] off topic: polinomio de taylor

2008-11-21 Por tôpico LEANDRO L RECOVA

Concordo com o Paulo. E um excelente livro e quando eu fiz Calculo na UnB nos 
anos 90, ele era adotado. > Date: Fri, 21 Nov 2008 19:18:08 -0200> From: [EMAIL 
PROTECTED]> To: obm-l@mat.puc-rio.br> Subject: Re: [obm-l] off topic: polinomio 
de taylor> > Olá Hermann e demais> colegas desta lista ... OBM-L,> ( escreverei 
sem acentos)> > O livro "Um Curso de Calculo", Volume 1, do Prof Hamilton 
Guidorizzi,> tem um capitulo - o cap. 15 - inteiramente dedicado ao polinomio 
de> Taylor. EM MINHA OPINIAO, este livro e, "ao menos", um dos melhores> dentre 
todos os livros de calculo escritos por brasileiros ...> muitissimo melhor que 
algumas porcarias estrangeiras adotadas em> conhecidas e boas Universidades e 
Faculdades do Brasil. E e muito> dificil nao entender o que e o polinomio de 
Taylor estudando por ele,> pois o autor e altamente didatico.> > Parece que foi 
ou ainda e adotado no ITA.> > EM MINHA OPINIAO, se voce quer estudar calculo 
seriamente e criar> alicerces seguros para um posterior aprofundamento, vale a 
pena ter os> 4 volumes e estudar por ele. ME PARECE que a sua fraqueza esta 
nos> exercicios, em pouca quantidade e triviais. Mas exercicios voce pega> em 
outros, ja classicos e bem conhecidos.> > Um Abracao> PSR, 62111081917> > > > > 
> 2008/11/19 Hermann <[EMAIL PROTECTED]>:> > Boa noite, gostaria de falar sobre 
3 assuntos:> >> > 1) Mais uma vez agradecer a todos que participam dessa lista, 
tirar dúvidas> > ou ler as dúvidas dos outros ensina bastante.> >> > 2) Estou 
achando que há algo de errado com o servidor pois não estou> > recebendo 
mensagens. (só consigo lendo no site)> >> > 3) Meu problema -> > (Preciso de 
ajuda para encontrar um texto (se possível em português) que> > explique 
Polinômios de Taylor. Nos livros que tenho de cálculo, o assunto é> > passado 
muito superficialmente.)> >> > Obrigado> > Hermann> > 
=> 
Instruções para entrar na lista, sair da lista e usar a lista em> 
http://www.mat.puc-rio.br/~obmlistas/obm-l.html> 
=

Re: [obm-l] Geometria de Superf�cies em R^3

2008-11-05 Por tôpico LEANDRO L RECOVA
Nao! Voce nao pode considerar que e o mesmo. Compare o valor da curvatura 
gaussiana da esfera (facil de calcular) com o valor da curvatura gaussiana 
da superficie M no mesmo ponto. Nao necessariamente e o mesmo.




From: [EMAIL PROTECTED]
Reply-To: obm-l@mat.puc-rio.br
To: obm-l@mat.puc-rio.br
Subject: Re: [obm-l] Geometria de Superfícies em R^3
Date: Tue, 4 Nov 2008 21:16:25 -0200

Oi,

Acompanhei a sua construção, mas meu problema está exatamente na parte
em que você parou. No livro do Manfredo, tem uma dica dizendo pra 
considerar

uma esfera em torno da superfície, e diminuí-la até que ela toque a
superfície em
um único ponto.
Você tornou precisa a idéia de que nesse ponto, os planos tangentes da
esfera
e da superfície coincidem, bem como o vetor normal (basta escolher uma
orientação
adequada da esfera).
Mas ainda não vejo como obter det( dN_p ) > 0 a partir disto.
Seria verdade que o dN da esfera e da superfície são o mesmo em p?

Valeu,

- Leandro.



=
Instruções para entrar na lista, sair da lista e usar a lista em
http://www.mat.puc-rio.br/~obmlistas/obm-l.html
=


RE: [obm-l] Geometria de Superf�cies em R^3

2008-11-04 Por tôpico LEANDRO L RECOVA
Acho que tem uma demonstracao desse problema no livro do Barret O'Neill. 
Deixa eu ver se lembro. Quando voce diz det(dNp) > 0 isso tambem quer dizer 
que sua superficie tem curvatura positiva (Lembre que a curvatura gaussiana 
e definida como o produto dos autovalores da aplicacao normal de Gauss). 
Entao, vamos provar a afirmativa de que para toda a superficie M em R^3 
compacta ha um ponto onde a curvatura gaussiana K e estritatamente positiva.


Seja M a superficie compacta em R^3 compacta, e considere f:M->R a funcao 
f(p)=||p||^2. Em termos de coordenadas, f(p)=sum(x_{i}^2). Agora, f e 
diferencial e portanto continua e M e compacta. Entao, f atinge seu ponto de 
maximo em algum ponto m de M. Observe que f mede a distancia da origem, 
entao m e simplesmente o ponto de maxima distancia da origem r=||m|| > 0. 
Isso quer dizer que M e tangente em p a esfera S de radio r e M esta dentro 
da esfera S. (Agora, deixo o resto com voce).


Regards,

Leandro
Los Angeles, CA.


From: [EMAIL PROTECTED]
Reply-To: obm-l@mat.puc-rio.br
To: obm-l@mat.puc-rio.br
Subject: [obm-l] Geometria de Superfícies em R^3
Date: Tue, 4 Nov 2008 18:16:11 -0200

Olá,

Gostaria de uma solução para o seguinte problema:

Toda superfície regular (de dimensão 2), compacta, em R^3 possui
um ponto elíptico, isto é, um ponto p tal que det( dN_p ) > 0, onde
dN_p é a derivada da aplicação normal de Gauss em p.

Este é o problema 16, seção 3-3 do livro do Manfredo, Differential
Geometry of Curves and Surfaces.

- Leandro.



=
Instruções para entrar na lista, sair da lista e usar a lista em
http://www.mat.puc-rio.br/~obmlistas/obm-l.html
=


Re: [obm-l] Re: Deriva��o impl�cita

2008-10-17 Por tôpico LEANDRO L RECOVA

Ok.

[(1+y')/2sqrt(x+1)]=[y'/2(sqrt(y))]

sqrt(x+y)*y'=(1+y').sqrt(y)

Agora, sqrt(x+)=sqrt(y)+1, entao,

(sqrt(y)+1)y'=(1+y')sqrt(y)

E a resposta segue, y'=sqrt(y). Era so voce ter isolado o y' na minha 
primeira resposta e substituir o valor original.


Got it?

Regards,

Leandro.



From: "Julio Sousa" <[EMAIL PROTECTED]>
Reply-To: obm-l@mat.puc-rio.br
To: obm-l@mat.puc-rio.br
Subject: Re: [obm-l] Re: Derivação implícita
Date: Fri, 17 Oct 2008 11:38:01 -0700

a resposta aqui é y' = sqrt(y)


2008/10/17 LEANDRO L RECOVA <[EMAIL PROTECTED]>

> Erro de digitacao: deveria ser sqrt(x+y) no primeiro termo.
>
>
>  From: "LEANDRO L RECOVA" <[EMAIL PROTECTED]>
>> Reply-To: obm-l@mat.puc-rio.br
>> To: obm-l@mat.puc-rio.br
>> Subject: RE: [obm-l] Re: Derivação implícita
>> Date: Fri, 17 Oct 2008 06:48:02 -0700
>>
>>
>> [(1+y')/2sqrt(x+1)]=[y'/2(sqrt(y))]
>>
>> Agora voce isola y'.
>>
>>
>>  From: "Julio Sousa" <[EMAIL PROTECTED]>
>>> Reply-To: obm-l@mat.puc-rio.br
>>> To: obm-l@mat.puc-rio.br
>>> Subject: [obm-l] Re: Derivação implícita
>>> Date: Fri, 17 Oct 2008 04:42:37 -0700
>>>
>>> derivar implícito sqrt(x+y) = sqrt(y) + 1 e achar y'. Alguém poderia 
me

>>> ajudar?
>>> On Fri, Oct 17, 2008 at 4:41 AM, Julio Sousa <[EMAIL PROTECTED]>
>>> wrote:
>>>
>>> > sqrt(x+y) = sqrt(y) + 1
>>> >
>>> >
>>> > --
>>> >
>>> >
>>> 


>>> > Atenciosamente
>>> > Júlio Alfredo Moreira Sousa Junior
>>> > Graduando em Ciência da Computação - UFJF
>>> >
>>> >
>>>
>>>
>>> --
>>>
>>> 


>>> Atenciosamente
>>> Júlio Alfredo Moreira Sousa Junior
>>> Graduando em Ciência da Computação - UFJF
>>> Telefones
>>> Comercial: (32) 3239-3312
>>> Celular: (32) 8847-8581
>>> Residencial: (32) 3217-2076
>>>
>>
>>
>> 
=

>> Instruções para entrar na lista, sair da lista e usar a lista em
>> http://www.mat.puc-rio.br/~obmlistas/obm-l.html
>> 
=

>>
>
>
> 
=

> Instruções para entrar na lista, sair da lista e usar a lista em
> http://www.mat.puc-rio.br/~obmlistas/obm-l.html
> 
=

>



--

Atenciosamente
Júlio Alfredo Moreira Sousa Junior
Graduando em Ciência da Computação - UFJF
Telefones
Comercial: (32) 3239-3312
Celular: (32) 8847-8581
Residencial: (32) 3217-2076



=
Instruções para entrar na lista, sair da lista e usar a lista em
http://www.mat.puc-rio.br/~obmlistas/obm-l.html
=


RE: [obm-l] Re: Deriva��o impl�cita

2008-10-17 Por tôpico LEANDRO L RECOVA

Erro de digitacao: deveria ser sqrt(x+y) no primeiro termo.



From: "LEANDRO L RECOVA" <[EMAIL PROTECTED]>
Reply-To: obm-l@mat.puc-rio.br
To: obm-l@mat.puc-rio.br
Subject: RE: [obm-l] Re: Derivação implícita
Date: Fri, 17 Oct 2008 06:48:02 -0700

[(1+y')/2sqrt(x+1)]=[y'/2(sqrt(y))]

Agora voce isola y'.



From: "Julio Sousa" <[EMAIL PROTECTED]>
Reply-To: obm-l@mat.puc-rio.br
To: obm-l@mat.puc-rio.br
Subject: [obm-l] Re: Derivação implícita
Date: Fri, 17 Oct 2008 04:42:37 -0700

derivar implícito sqrt(x+y) = sqrt(y) + 1 e achar y'. Alguém poderia me
ajudar?
On Fri, Oct 17, 2008 at 4:41 AM, Julio Sousa <[EMAIL PROTECTED]> 
wrote:


> sqrt(x+y) = sqrt(y) + 1
>
>
> --
>
> 


> Atenciosamente
> Júlio Alfredo Moreira Sousa Junior
> Graduando em Ciência da Computação - UFJF
>
>


--

Atenciosamente
Júlio Alfredo Moreira Sousa Junior
Graduando em Ciência da Computação - UFJF
Telefones
Comercial: (32) 3239-3312
Celular: (32) 8847-8581
Residencial: (32) 3217-2076



=
Instruções para entrar na lista, sair da lista e usar a lista em
http://www.mat.puc-rio.br/~obmlistas/obm-l.html
=



=
Instruções para entrar na lista, sair da lista e usar a lista em
http://www.mat.puc-rio.br/~obmlistas/obm-l.html
=


RE: [obm-l] Re: Deriva��o impl�cita

2008-10-17 Por tôpico LEANDRO L RECOVA

[(1+y')/2sqrt(x+1)]=[y'/2(sqrt(y))]

Agora voce isola y'.



From: "Julio Sousa" <[EMAIL PROTECTED]>
Reply-To: obm-l@mat.puc-rio.br
To: obm-l@mat.puc-rio.br
Subject: [obm-l] Re: Derivação implícita
Date: Fri, 17 Oct 2008 04:42:37 -0700

derivar implícito sqrt(x+y) = sqrt(y) + 1 e achar y'. Alguém poderia me
ajudar?
On Fri, Oct 17, 2008 at 4:41 AM, Julio Sousa <[EMAIL PROTECTED]> 
wrote:


> sqrt(x+y) = sqrt(y) + 1
>
>
> --
>
> 


> Atenciosamente
> Júlio Alfredo Moreira Sousa Junior
> Graduando em Ciência da Computação - UFJF
>
>


--

Atenciosamente
Júlio Alfredo Moreira Sousa Junior
Graduando em Ciência da Computação - UFJF
Telefones
Comercial: (32) 3239-3312
Celular: (32) 8847-8581
Residencial: (32) 3217-2076



=
Instruções para entrar na lista, sair da lista e usar a lista em
http://www.mat.puc-rio.br/~obmlistas/obm-l.html
=


RE: [obm-l] RE: [obm-l] S�rie de Laurent - Ajuda, por favor?

2008-10-12 Por tôpico LEANDRO L RECOVA
Se o exercicio esta pedindo pra desenvolver em torno de z=i, entao, voce nao 
toca em g(z) e desenvolve a serie de Laurent para 1/(z-i). E ai multiplica a 
serie pelo termo em colchetes de f(z).




From: César Santos <[EMAIL PROTECTED]>
Reply-To: obm-l@mat.puc-rio.br
To: obm-l@mat.puc-rio.br
Subject: [obm-l] RE: [obm-l] Série de Laurent - Ajuda, por favor?
Date: Sun, 12 Oct 2008 13:54:37 -0700 (PDT)

Muito obrigado pela atenção, felizmente eu consegui desenrolar a questão, 
mas agora estou com dúvida quanto a outra série f(z) = 1/[z(z²+1)] em torno 
da singularidade z= i. A minha dúvida é se eu posso fazer f(z) = 1/(z-i) * 
[1/[z*(z+i)], desenrolar a série em potências de (z-i) para a função g(z) = 
1/[z*(z+i)] e depois multiplicar o resultado por 1/(z-i). Se for possível 
eu terei resolvido a questão, caso contrário...

 
--- Em dom, 12/10/08, LEANDRO L RECOVA <[EMAIL PROTECTED]> escreveu:

De: LEANDRO L RECOVA <[EMAIL PROTECTED]>
Assunto: RE: [obm-l] Série de Laurent - Ajuda, por favor?
Para: obm-l@mat.puc-rio.br
Data: Domingo, 12 de Outubro de 2008, 17:30

Tente fazer u=z-1. Entao, pela definicao, 1/u < 1. Agora, substitua na
serie,

f(z)= 1/z - 1/z^2 = 1/z(1-1/z)

f(u) = (1/u+1)(1-1/(u+1))

Agora, repare que

1/(u+1) = 1/u(1+1/u) = (1/u)[(1-(1/u)+(1/u)^2 + ] =
sum(n=1)(infty)(-1)^(n)* (1/u)^(n).

Substitua isso em f(u) agora,

f(u)= 1(u+1) [1 - sum_{n=0}^{\infty}(-1)^(n)* (1/u)^(n)]

Como estamos tirando 1 da soma, e temos o sinal de (-), todos termos trocam
de sinal, entao (-1)^n becomes (-1)^(n+1) e o somatorio comeca por n=1,

f(u) = 1/(u+1)[ sum _{n=1}^{\infty}(-1)^(n+1)*(1/u)^(n)

f(z)=1/z [ sum_{n=1}^{\infty}(-1)^(n+1)*(z-1)*(-n)]

Encontrei a resposta diferente, mas tente fazer de novo. Eu nao tinha lapis
e caneta aqui. Estou num Starbucks aqui em Irvine, California. Mas, a
solucao e por ai. Sempre que tiver isso, faca uma substituicao do tipo que
eu fiz, pois voce tem que encaixar o resultado da soma geometrica infinita
sempre que a razao q < 1.

Leandro.

>From: César Santos <[EMAIL PROTECTED]>
>Reply-To: obm-l@mat.puc-rio.br
>To: obm-l@mat.puc-rio.br
>Subject: [obm-l] Série de Laurent - Ajuda, por favor?
>Date: Sun, 12 Oct 2008 07:19:45 -0700 (PDT)
>
>Determinar a série de Laurent no domínio |z-1| > 1
>para f(z) = (z-1)/z²
>Poderia explicar passo a passo a resolução?
>A resposta é somatório, com n variando de 1 ao infinito, de
>(-1)^(n+1)*n*(z-1)^(-n)
>onde a^b significa 'a' elevado a 'b'  e * indica
multiplicação.
>
>
>   Novos endereços, o Yahoo! que você conhece. Crie um email novo com
a
>sua cara @ymail.com ou @rocketmail.com.
>http://br.new.mail.yahoo.com/addresses


=
Instruções para entrar na lista, sair da lista e usar a lista em
http://www.mat.puc-rio.br/~obmlistas/obm-l.html
=



  Novos endereços, o Yahoo! que você conhece. Crie um email novo com a 
sua cara @ymail.com ou @rocketmail.com.

http://br.new.mail.yahoo.com/addresses



=
Instruções para entrar na lista, sair da lista e usar a lista em
http://www.mat.puc-rio.br/~obmlistas/obm-l.html
=


RE: [obm-l] S�rie de Laurent - Ajuda, por favor?

2008-10-12 Por tôpico LEANDRO L RECOVA
Tente fazer u=z-1. Entao, pela definicao, 1/u < 1. Agora, substitua na 
serie,


f(z)= 1/z - 1/z^2 = 1/z(1-1/z)

f(u) = (1/u+1)(1-1/(u+1))

Agora, repare que

1/(u+1) = 1/u(1+1/u) = (1/u)[(1-(1/u)+(1/u)^2 + ] = 
sum(n=1)(infty)(-1)^(n)* (1/u)^(n).


Substitua isso em f(u) agora,

f(u)= 1(u+1) [1 - sum_{n=0}^{\infty}(-1)^(n)* (1/u)^(n)]

Como estamos tirando 1 da soma, e temos o sinal de (-), todos termos trocam 
de sinal, entao (-1)^n becomes (-1)^(n+1) e o somatorio comeca por n=1,


f(u) = 1/(u+1)[ sum _{n=1}^{\infty}(-1)^(n+1)*(1/u)^(n)

f(z)=1/z [ sum_{n=1}^{\infty}(-1)^(n+1)*(z-1)*(-n)]

Encontrei a resposta diferente, mas tente fazer de novo. Eu nao tinha lapis 
e caneta aqui. Estou num Starbucks aqui em Irvine, California. Mas, a 
solucao e por ai. Sempre que tiver isso, faca uma substituicao do tipo que 
eu fiz, pois voce tem que encaixar o resultado da soma geometrica infinita 
sempre que a razao q < 1.


Leandro.


From: César Santos <[EMAIL PROTECTED]>
Reply-To: obm-l@mat.puc-rio.br
To: obm-l@mat.puc-rio.br
Subject: [obm-l] Série de Laurent - Ajuda, por favor?
Date: Sun, 12 Oct 2008 07:19:45 -0700 (PDT)

Determinar a série de Laurent no domínio |z-1| > 1
para f(z) = (z-1)/z²
Poderia explicar passo a passo a resolução?
A resposta é somatório, com n variando de 1 ao infinito, de 
(-1)^(n+1)*n*(z-1)^(-n)

onde a^b significa 'a' elevado a 'b'  e * indica multiplicação.


  Novos endereços, o Yahoo! que você conhece. Crie um email novo com a 
sua cara @ymail.com ou @rocketmail.com.

http://br.new.mail.yahoo.com/addresses



=
Instruções para entrar na lista, sair da lista e usar a lista em
http://www.mat.puc-rio.br/~obmlistas/obm-l.html
=


RE: [obm-l] Equa��es diferenciais

2008-10-11 Por tôpico LEANDRO L RECOVA

Faca u=y' e considerando u diferente de zero,

u'+t(u)^2=0
u'/u^2 = -t

Integre ambos os lados,

Int (du/u^2)=-int(t)dt

-1/u = -t^2/2 + C1

Substitua o valor de u,

-1/y' = -t^2/2 + C1

y' = 1/(C1-t^2/2)

Agora, voce analisa os casos de C1 e deve chegar ao resultado desejado.

Leandro







From: warley ferreira <[EMAIL PROTECTED]>
Reply-To: obm-l@mat.puc-rio.br
To: Lista de Discussão 
Subject: [obm-l] Equações diferenciais
Date: Sat, 11 Oct 2008 07:32:34 -0700 (PDT)

Peço ajuda nesta E.D.O de 2ªOrdem
Equações sem a variável dependente:
      y ,, + t ( y , ) 2 = 0
 
Faço a mudança de variável e chego numa equação separável , mas não consigo 
chegar nesta resposta.

 

Resposta: y = ( 1/k) ln | (k-t) / (k+t) | + c2 se c1 = k2 > 0
 
                y = (2/k) arc tg (t/k) + c2 se  c1 = - k2 < 0
                
               y = -2 t –1 + c2 se c1  = 0
          
               y = c
Desde já obrigado!

Warley Souza


  Novos endereços, o Yahoo! que você conhece. Crie um email novo com a 
sua cara @ymail.com ou @rocketmail.com.

http://br.new.mail.yahoo.com/addresses



=
Instruções para entrar na lista, sair da lista e usar a lista em
http://www.mat.puc-rio.br/~obmlistas/obm-l.html
=


RE: [obm-l] uma luz por favor

2008-10-02 Por tôpico LEANDRO L RECOVA

Marcelo,

Acho que voce deve considerar o total de possibilidades e subtrair os casos 
em que elas aparecem juntas.


SS 5 4 3 2 1 = > 3*2*5*(4!)

Total = 7! - 5*(3!)*(4!) = 7!-5*(3*2*1*4*3*2*1) = 7! - 6!.


Regards,

Leandro.
Los Angeles, California.


From: "Marcelo Costa" <[EMAIL PROTECTED]>
Reply-To: obm-l@mat.puc-rio.br
To: obm-l@mat.puc-rio.br
Subject: [obm-l] uma luz por favor
Date: Thu, 2 Oct 2008 18:08:31 -0300

Vejamos o problema:
Uma CPI vai interrogar 3 secretárias, 2 empresários e 2 motoristas, de
quantas maneiras distintas ela pode fazer o interrogatório de modo que não
haja interrogatórios consecutivos das secretárias?
R 7! - 6!
Até pense em considerar duas secretárias como uma, pois qdo conto duas
estarei eliminando as 3, mas se considerar 1 e 2 como uma não terei a
seqüência 132 então? até pensei no lema de kaplansky mas não bate.



=
Instruções para entrar na lista, sair da lista e usar a lista em
http://www.mat.puc-rio.br/~obmlistas/obm-l.html
=


RE: [obm-l] Probabilidade!

2008-10-02 Por tôpico LEANDRO L RECOVA

Acho que voce resolve isso usando a distribuicao binomial.





From: jose silva <[EMAIL PROTECTED]>
Reply-To: obm-l@mat.puc-rio.br
To: 
Subject: [obm-l] Probabilidade!
Date: Thu, 2 Oct 2008 02:45:49 +





Em uma escola é feita uma atividade lúdica, envolvendo a aplicação de 
probabilidades. Durante a aula, coloca-se dentro de uma urna, 25 bolas 
marcadas com os números de 1 a 25. Em seguida, são distribuídos entre os 
alunos cartelas contendo estes números, em ordem crescente, ou seja, do 
número 1 ao 25. Após isso, pedem-se aos alunos para marcarem 15 números 
aleatoriamente, nesta cartela. Feito isso, qual a probabilidade de após a 
retirada aleatória e sem reposição, de 15 bolas consecutivas desta urna, de 
um dos estudantes acertar os 15 números? De um dos alunos acertar 14 ou 13 
ou 12 ou  11 números?




Notícias direto do New York Times, gols do Lance, videocassetadas e muitos 
outros vídeos no MSN Videos! Confira já!

_
Cansado de espaço para só 50 fotos? Conheça o Spaces, o site de 
relacionamentos com até 6,000 fotos!

http://www.amigosdomessenger.com.br



=
Instruções para entrar na lista, sair da lista e usar a lista em
http://www.mat.puc-rio.br/~obmlistas/obm-l.html
=


RE: [obm-l] Indica��o de livro - tensores

2008-09-25 Por tôpico LEANDRO L RECOVA
O Elon Lages Lima tinha um livro de tensores muito bom que se chamava 
Analise Tensorial.




From: César Santos <[EMAIL PROTECTED]>
Reply-To: obm-l@mat.puc-rio.br
To: obm-l@mat.puc-rio.br
Subject: [obm-l] Indicação de livro -  tensores
Date: Tue, 23 Sep 2008 18:01:35 -0700 (PDT)

Olá, alguém poderia me indicar um bom livro sobre tensores para alguém que 
está iniciando agora nesse assunto?



  Novos endereços, o Yahoo! que você conhece. Crie um email novo com a 
sua cara @ymail.com ou @rocketmail.com.

http://br.new.mail.yahoo.com/addresses



=
Instruções para entrar na lista, sair da lista e usar a lista em
http://www.mat.puc-rio.br/~obmlistas/obm-l.html
=


RE: [obm-l] Teoria dos Conjuntos

2008-09-03 Por tôpico LEANDRO L RECOVA
No meu ponto de vista, se { }  representasse o conjunto vazio eu 
consideraria falsa.




From: "Luiz Rodrigues" <[EMAIL PROTECTED]>
Reply-To: obm-l@mat.puc-rio.br
To: obm-l@mat.puc-rio.br
Subject: [obm-l] Teoria dos Conjuntos
Date: Wed, 3 Sep 2008 14:00:02 -0300

Olá pessoal!!!
Tudo bem???
Um aluno me apresentou uma senteça que, segundo um outro professor, é
verdadeira.
A sentença é:

"x pertence { } -> x é verde"

Na minha opinião, esta sentença é falsa, porque "x pertence { }" é falsa.
Segundo o meu aluno, o que o outro professor alegou é que "x pode ser
qualquer coisa".
O que vocês acham???
Muito obrigado!!!
Abração para todos!!!
Luiz.

=
Instruções para entrar na lista, sair da lista e usar a lista em
http://www.mat.puc-rio.br/~obmlistas/obm-l.html
=



=
Instruções para entrar na lista, sair da lista e usar a lista em
http://www.mat.puc-rio.br/~obmlistas/obm-l.html
=


Re: [obm-l] Transformada de Fourier

2008-08-20 Por tôpico LEANDRO L RECOVA
De uma olhada no livro "Ten lectures on Wavelets" (Ingrid Daubechies) ou do 
G.Strang "Filter Banks and Wavelets". Eles tem um capitulo sobre como 
determinar Wavelets que tem suporte compacto. Voce tambem tem que impor 
condicoes nos extremos (vanishing moments).




From: "Rafael Ando" <[EMAIL PROTECTED]>
Reply-To: obm-l@mat.puc-rio.br
To: obm-l@mat.puc-rio.br
Subject: Re: [obm-l] Transformada de Fourier
Date: Tue, 19 Aug 2008 22:28:31 +0200

Ola... eu tenho apenas o valor absoluto de A mesmo, mas para a transformada
inversa eu posso encontrar qualquer uma que dê um resultado compatível,
desde que seja x seja real... será que mesmo assim não tem como eu achar?

Eu estou fazendo um programa para analisar wavelets e tenho que encontrar a
wavelet cuja ampiltude (ie, o valor absoluto da transformada) seja um
trapézio. Eu quero que a minha wavelet seja par (mas se eu encontrar por
algum método qualquer wavelet que não-par é facil defasar) e, claro, real.
Aqui no meu estágio sugeriram que eu usasse:

x[i] = constante * somatório em j de ( cos(2*pi*freq_j*t_i) * A[j] )
onde freq_j = freq0 + j*delta f, t_i = Tmin + i*dt
x[i] seria o "valor" da wavelet no instante t_i e A[j] a amplitude para
frequencia freq_j a constante vale 2g/N, ond g é o ganho, um parametro
do programa... bom, não é importante.

Mas essa sugestão não funcionou... eu confesso que não entendo muito de
transformada de Fourier (quando eu peguei esse estágio só me falaram que
precisava saber Java...0_o), e o pessoal aqui tá muito ocupado pra poder me
ajudar, por isso resolvi pedir ajuda pra voces :)

Obrigado pela ajuda!

On Tue, Aug 19, 2008 at 7:24 PM, LEANDRO L RECOVA 
<[EMAIL PROTECTED]>wrote:


> Pra voce encontrar a transformada inversa, voce teria que ter informacao
> sobre a fase. Voce tem certeza que nao tem o vetor A complexo?
>
>
>
>  From: "Rafael Ando" <[EMAIL PROTECTED]>
>> Reply-To: obm-l@mat.puc-rio.br
>> To: obm-l@mat.puc-rio.br
>> Subject: [obm-l] Transformada de Fourier
>> Date: Tue, 19 Aug 2008 15:27:42 +0200
>>
>>
>> Ola amigos eu estou com um problema
>>
>> Eu tenho uma sequencia discreta A que representa o valor absoluto da
>> transformada de Fourier de uma sequencia x, que eu nao conheco. Eu 
tenho

>> que
>> encontrar x, e tenho que por hipotese x eh uma sequencia real (A 
satisfaz
>> A[k] = A[N-k], condicao necessaria pra x ser real, sem problemas). Como 
A

>> eh
>> apenas o valor absoluto da transformada de x, eh possivel que existam
>> varios
>> x, qualquer x possivel serve...
>>
>> Como eu posso proceder?
>>
>> --
>> Rafael
>>
>
>
> 
=

> Instruções para entrar na lista, sair da lista e usar a lista em
> 
http://www.mat.puc-rio.br/~obmlistas/obm-l.html<http://www.mat.puc-rio.br/%7Eobmlistas/obm-l.html>
> 
=

>



--
Rafael



=
Instruções para entrar na lista, sair da lista e usar a lista em
http://www.mat.puc-rio.br/~obmlistas/obm-l.html
=


RE: [obm-l] Transformada de Fourier

2008-08-19 Por tôpico LEANDRO L RECOVA
Pra voce encontrar a transformada inversa, voce teria que ter informacao 
sobre a fase. Voce tem certeza que nao tem o vetor A complexo?





From: "Rafael Ando" <[EMAIL PROTECTED]>
Reply-To: obm-l@mat.puc-rio.br
To: obm-l@mat.puc-rio.br
Subject: [obm-l] Transformada de Fourier
Date: Tue, 19 Aug 2008 15:27:42 +0200

Ola amigos eu estou com um problema

Eu tenho uma sequencia discreta A que representa o valor absoluto da
transformada de Fourier de uma sequencia x, que eu nao conheco. Eu tenho 
que

encontrar x, e tenho que por hipotese x eh uma sequencia real (A satisfaz
A[k] = A[N-k], condicao necessaria pra x ser real, sem problemas). Como A 
eh
apenas o valor absoluto da transformada de x, eh possivel que existam 
varios

x, qualquer x possivel serve...

Como eu posso proceder?

--
Rafael



=
Instruções para entrar na lista, sair da lista e usar a lista em
http://www.mat.puc-rio.br/~obmlistas/obm-l.html
=


RE: [obm-l] Fun��o sobrejetiva...

2008-08-16 Por tôpico LEANDRO L RECOVA

Ruy ,

A solucao esta correta. Eles usaram a definicao de funcao sobrejetiva e 
provaram que a cada y da imagem da funcao existe um x no dominio de f. Voce 
afirmou a bijetividade, mas e algo que pode ser facilmente provado tambem.


Leandro.



From: "ruy de oliveira souza" <[EMAIL PROTECTED]>
Reply-To: obm-l@mat.puc-rio.br
To: obm-l@mat.puc-rio.br
Subject: [obm-l] Função sobrejetiva...
Date: Sat, 16 Aug 2008 14:48:07 -0300

Uma questão do Ita de 2005 tem o seguinte enunciado...Seja
f:lR-{1}>lR-{1}, definida por f(x)=(x+1)/(x-1). Um dos itens  questiona
se ela é sobrejetiva ou não. Dá pra se provar facilmente fazendo o gráfico
que ela é. Pode-se também calcular os limites da função fazendo x tender
a mais e menos infinito, além dos limites laterais quando x tende a 1. O 
que

me chama a atenção nessa questão foi a resolução feita por um cursinho
famoso de SP. Gostaria de saber se vocês concordam com essa resolução que
está na internet. Eles resolvem da seguinte maneira: fazem f(x)=y. Dai tem 
a

equação y=(x+1)/(x-1). Isolam x e obtem x=(y+1)/(y-1). Argumentam que para
todo y pertencente a lR-{1} existe x pertencente a lR-{1} tal que y=f(x).
Mas isso é a função inversa, obtida a partir do fato de se saber que a
função é bijetora. Ou seja, posso fazer dessa maneira se souber de antemão
se ela é bijetora. Então , na verdade não estou provando sobrejetividade
alguma. O que vocês acham? Corrijam-me se eu estiver enganado.
  Abraços



=
Instruções para entrar na lista, sair da lista e usar a lista em
http://www.mat.puc-rio.br/~obmlistas/obm-l.html
=


RE: [obm-l] An�lise

2008-07-21 Por tôpico LEANDRO L RECOVA
Tenta usar o teorema do valor medio no segundo membro e use a hipotese do 
teorema. Acho que isso prova o teorema.




From: "Alexsandro Néo." <[EMAIL PROTECTED]>
Reply-To: obm-l@mat.puc-rio.br
To: obm-l@mat.puc-rio.br
Subject: [obm-l] Análise
Date: Mon, 21 Jul 2008 22:27:52 -0300

Alguém ajuda?

Se f : Rm --> Rn é diferenciável, com lim f ´ (x) . x = 0 (quando módulo de 
x tende para o infinito) então a aplicação g : Rm --> Rn, definida por g(x) 
= f(2x) - f(x), é limitada.


obrigado desde já.
=
Instruções para entrar na lista, sair da lista e usar a lista em
http://www.mat.puc-rio.br/~obmlistas/obm-l.html
=



=
Instruções para entrar na lista, sair da lista e usar a lista em
http://www.mat.puc-rio.br/~obmlistas/obm-l.html
=


RE: [obm-l] D�vida �lgebra Linear [ URGENTE ]

2008-07-08 Por tôpico LEANDRO L RECOVA

O que voce esta chamando de P3(t,R)



From: "Hugo Henley" <[EMAIL PROTECTED]>
Reply-To: obm-l@mat.puc-rio.br
To: 
Subject: [obm-l] Dúvida Álgebra Linear [ URGENTE ]
Date: Tue, 8 Jul 2008 16:53:06 -0300

Alguém poderia me ajudar a resolver a seguinte questão ?



Seja T: R4  -> P3(t,R) dado por T(a,b,c,d) = at² + (b-c)t + d





a)  Determine KerT, ImT e explicite uma base para cada um desses
subespaços.

b)  Descreva geometricamente os subespaços do item anterior como
subespaços de R3.





Obrigado,



Hugo Henley




=
Instruções para entrar na lista, sair da lista e usar a lista em
http://www.mat.puc-rio.br/~obmlistas/obm-l.html
=


RE: [obm-l] Fun��es - ITA 1978

2008-05-02 Por tôpico LEANDRO L RECOVA

Igor,

O enunciado esta correto? Parece que a frase

"Se B
está contido em R e o conjunto f^-1(B) = { x pertence a R ; f(x)
pertence a B }, "

esta incompleta. Voce esta dizendo que f-1(B) tambem esta em R?





From: "Igor Battazza" <[EMAIL PROTECTED]>
Reply-To: obm-l@mat.puc-rio.br
To: obm-l@mat.puc-rio.br
Subject: [obm-l] Funções - ITA 1978
Date: Fri, 2 May 2008 12:14:48 -0300

Olá, alguem poderia me dar uma ajuda na explicação dessa questão, pois
eu cheguei em um resultado proximo, mas de uma maneira meio mistica,
chutando (ou seja, de uma forma incorreta). Lá vai:

Sejam R o conjunto dos números reais e f uma função de R em R. Se B
está contido em R e o conjunto f^-1(B) = { x pertence a R ; f(x)
pertence a B }, então:

a) f[ f^-1(B) ] está contido em B; (Alternativa correta, mas pouco me
importa :P )
b) f[ f^-1(B) ] = B se f é injetora;
c) f[ f^-1(B) ] = B
d) f^-1[ f(B) ] = B se f é sobrejetora;
e) n.d.a.

OBS: f^-1 é a inversa de f.

Obrigado desde já!

=
Instruções para entrar na lista, sair da lista e usar a lista em
http://www.mat.puc-rio.br/~obmlistas/obm-l.html
=



=
Instruções para entrar na lista, sair da lista e usar a lista em
http://www.mat.puc-rio.br/~obmlistas/obm-l.html
=


RE: [obm-l] Teoria da Medida

2008-01-22 Por tôpico LEANDRO L RECOVA

Olhe nessa pagina:

http://ocw.mit.edu/OcwWeb/Mathematics/index.htm

O livro do Zigmund e muito usado, o do Rudin, e eu tambem gostei do Royden.

Saudacoes rubro-negras,

Leandro.


From: "Valdoir Wathier" <[EMAIL PROTECTED]>
Reply-To: obm-l@mat.puc-rio.br
To: obm-l@mat.puc-rio.br
Subject: [obm-l] Teoria da Medida
Date: Tue, 22 Jan 2008 18:06:44 -0300

Alguém sabe me indicar algum material (preferencialmente on-line) sobre
Teoria da Medida?

Abraço a todos,

Valdoir Wathier



=
Instruções para entrar na lista, sair da lista e usar a lista em
http://www.mat.puc-rio.br/~obmlistas/obm-l.html
=


RE: [obm-l] off-topic: (rela��o entre derivabilidade e integrabilidade)

2008-01-16 Por tôpico LEANDRO L RECOVA

Cabri,

Se voce considera f:[a,b] C R -> R uma funcao derivavel em [a,b], entao, f e 
continua. Agora, se voce quiser ir mais adiante, se voce impor que f tambem 
e limitada, entao, voce garante a integrabilidade dela no intervalo [a,b]. 
Nao entrei em detalhes, mas se voce quiser, me envie um email.


Claro, ha outros casos a considerar tal como a integrabilidade em toda a 
reta, integrabilidade no sentido de Lebesgue, integrabilidade em R^n, etc. 
Eu so escrevi algo geral para o caso de dimensao 1.




Saudacoes,

Leandro.



From: "Tio Cabri st" <[EMAIL PROTECTED]>
Reply-To: obm-l@mat.puc-rio.br
To: 
Subject: [obm-l] off-topic: (relação entre derivabilidade e  
integrabilidade)  Date: Wed, 16 Jan 2008 07:49:46 -0200


Amigos, bom dia. Antes de incomodá-los com mais uma dúvida (dívida),
quero agradecer a todos os que participam dela. Lendo a lista aprendo 
muito.


Agora a minha d(Í)vida:

Quando uma função é derivável, o que posso dizer sobre ser
integrável?
Eu acho que não posso afirmar nada, mas não sei dar um  exemplo (ou
contra-exemplo).
Obrigado
Cabri







=
Instruções para entrar na lista, sair da lista e usar a lista em
http://www.mat.puc-rio.br/~obmlistas/obm-l.html
=



=
Instruções para entrar na lista, sair da lista e usar a lista em
http://www.mat.puc-rio.br/~obmlistas/obm-l.html
=


Re: [obm-l] cossenos sucessivos

2007-12-20 Por tôpico LEANDRO L RECOVA
O livro do Rudin "Principles of Mathematical Analysis" apresenta o principio 
da contracao e o teorema do ponto fixo no capitulo 9. Inclusive, a 
demonstracao e uma maneira de construir tal ponto. De uma olhada, e bem 
interessante.


Leandro.



From: "Bruno França dos Reis" <[EMAIL PROTECTED]>
Reply-To: obm-l@mat.puc-rio.br
To: obm-l@mat.puc-rio.br
Subject: Re: [obm-l] cossenos sucessivos
Date: Thu, 20 Dec 2007 19:26:34 +0100

Ola.
Este problema ja foi (bem) discutido pelo menos duas vezes aqui na lista.
Uma delas foi, se não me engano, no segundo semestre de 2005. Nessa
discussão, demonstrei que a sequencia a_n = f(a_(n-1)) converge para um
ponto fixo de f.

De uma olhada nos arquivos!

Bruno

2007/12/20, Ojesed Mirror <[EMAIL PROTECTED]>:
>
>  Numa calculadora operando em radianos, submete-se qualquer número a
> operações sucessivas de cosseno e observa-se que o número converge para 
um

> mesmo valor, independente do valor inicial adotado.
> O valor pode ser obtido numericamente de cos(x)=x.
>
> Mostrar que esta operação transforma qualquer real neste número.
>
> Ojesed
>



--
Bruno FRANÇA DOS REIS

msn: [EMAIL PROTECTED]
skype: brunoreis666
tel: +33 (0)6 28 43 42 16

e^(pi*i)+1=0



=
Instruções para entrar na lista, sair da lista e usar a lista em
http://www.mat.puc-rio.br/~obmlistas/obm-l.html
=


RE: [obm-l] an�lise complexa

2007-12-07 Por tôpico LEANDRO L RECOVA

Voce tentou usar o principio do Maximo?

Leandro.



From: "Kleber Bastos" <[EMAIL PROTECTED]>
Reply-To: obm-l@mat.puc-rio.br
To: obm-l@mat.puc-rio.br
Subject: [obm-l] análise complexa
Date: Fri, 7 Dec 2007 11:36:35 -0200

Seja f: U --> C ( complexos ) uma função holomorfa, onde U é um
domínio.Suponha que exista um ponto (a) pertencente a U tal que
|f(a)|<=|f(z)| para todo ponto z pertencente a U. Mostre que , ou bem
f(a)=0, ou bem f é uma função constante.

--
Kleber B. Bastos



=
Instruções para entrar na lista, sair da lista e usar a lista em
http://www.mat.puc-rio.br/~obmlistas/obm-l.html
=


RE: [obm-l] integral simples

2007-12-03 Por tôpico LEANDRO L RECOVA
Voce quer saber a primitiva ou e uma integral definida? Se for definida, 
quais sao os limites de integracao?




From: Klaus Ferraz <[EMAIL PROTECTED]>
Reply-To: obm-l@mat.puc-rio.br
To: obm-l@mat.puc-rio.br
Subject: [obm-l] integral simples
Date: Sat, 1 Dec 2007 17:47:41 -0800 (PST)

Olá alguem sabe como que resolvo a seguinte integral:
$e^x / x.


  Abra sua conta no Yahoo! Mail, o único sem limite de espaço para 
armazenamento!

http://br.mail.yahoo.com/



=
Instruções para entrar na lista, sair da lista e usar a lista em
http://www.mat.puc-rio.br/~obmlistas/obm-l.html
=


RE: [obm-l] off: Livro de An�lise II do Elon

2007-11-30 Por tôpico LEANDRO L RECOVA
Outro livro interessante tambem seria do Walter Rudin: "Principles of 
Mathematical Analysis.". Tem uma otima teoria e bons exercicios tambem. As 
vezes, estudar somente por um livro, pode desanimar uma pessoa. Tente outros 
tambem e mantenha a mente aberta.


Regards,

Leandro.



From: Francisco <[EMAIL PROTECTED]>
Reply-To: obm-l@mat.puc-rio.br
To: 
Subject: RE: [obm-l] off: Livro de Análise II do Elon
Date: Fri, 30 Nov 2007 18:35:13 +0300


seria ótimo para nós estudantes <> em análise!> Date: Fri, 30 Nov 
2007 12:54:30 -0200> Subject: Re: [obm-l] off: Livro de Análise II do Elon> 
From: [EMAIL PROTECTED]> To: obm-l@mat.puc-rio.br> > > Oi Francisco.> 
Eu tenho boa parte dos exercicios do livro de analise vol 2 do elon> 
resolvidos e tenho um plano de um dia escrever um livro com esse material.> 
Se alguem não publicar antes de mim, acho que vai ser muito util aos> 
alunos de analise.> > Abs.> > Rivaldo> > Olá!> >> > Desde minha graduação 
(em matemática) que estudo análise pelos livros do> > Elon. Em particular o 
<< Curso de Análise - Vol. II >>. Este último possui> > uma <> 
de problemas. No entanto, até hoje há muitos e muitos> > exercícios que não 
consegui resolver. Assim, gostaria de saber se alguém> > aqui na lista 
conhece algum manual contendo soluções/sugestões para> > <> 
problemas daquele livro.> >> > Agradeço desde já e peço desculpas pelo 
<>,> > Francisco> > 
_> > Veja 
mapas e encontre as melhores rotas para fugir do trânsito com o Live> > 
Search Maps!> > http://www.livemaps.com.br/index.aspx?tr=true> > > 
=> 
Instruções para entrar na lista, sair da lista e usar a lista em> 
http://www.mat.puc-rio.br/~obmlistas/obm-l.html> 
=

_
Receba GRÁTIS as mensagens do Messenger no seu celular quando você estiver 
offline. Conheça  o MSN Mobile!

http://mobile.live.com/signup/signup2.aspx?lc=pt-br



=
Instruções para entrar na lista, sair da lista e usar a lista em
http://www.mat.puc-rio.br/~obmlistas/obm-l.html
=


RE: [obm-l] off: Livro de An�lise II do Elon

2007-11-30 Por tôpico LEANDRO L RECOVA

Francisco,

Eu nao conheco tal livro. Mas, eu estava lendo na ultima edicao da revista 
NOTICES da AMS, onde eles mostram uma bibliografia do Kaplansky e 
depoimentos de outros matematicos, e Kaplansky dizia que as vezes a gente 
tem que insistir num problema, dias, anos, etc. Quando voce estiver cansado 
do problema, estude outra coisa diferente, e volte nele depois.  Isso e 
matematica.




Regards,



From: Francisco <[EMAIL PROTECTED]>
Reply-To: obm-l@mat.puc-rio.br
To: Lista de discursão 
Subject: [obm-l] off: Livro de Análise II do Elon
Date: Fri, 30 Nov 2007 15:12:35 +0300


Olá!

Desde minha graduação (em matemática) que estudo análise pelos livros do 
Elon. Em particular o << Curso de Análise - Vol. II >>. Este último possui 
uma <> de problemas. No entanto, até hoje há muitos e muitos 
exercícios que não consegui resolver. Assim, gostaria de saber se alguém 
aqui na lista conhece algum manual contendo soluções/sugestões para 
<> problemas daquele livro.


Agradeço desde já e peço desculpas pelo <>,
  Francisco
_
Veja mapas e encontre as melhores rotas para fugir do trânsito com o Live 
Search Maps!

http://www.livemaps.com.br/index.aspx?tr=true



=
Instruções para entrar na lista, sair da lista e usar a lista em
http://www.mat.puc-rio.br/~obmlistas/obm-l.html
=


RE: [obm-l] complexos_demonstra�

2007-11-30 Por tôpico LEANDRO L RECOVA
|(1 + i)z³ + iz| < |sqrt(2)|(1/8) + (1/2) =sqrt(2/64) + 1/2 =sqrt(1/32)+1/2 
= (1/sqrt(2))*1/4 + 1/2 < 1/4 + 1/2 = 3/4.


Na ultima desigualdade, use o fato de que 1/sqrt(2) < 1.

Regards.


From: "Ney Falcao" <[EMAIL PROTECTED]>
Reply-To: obm-l@mat.puc-rio.br
To: obm-l@mat.puc-rio.br
Subject: [obm-l] complexos_demonstração
Date: Thu, 29 Nov 2007 20:56:05 -0200

Olá pessoal,

Poderiam me ajudar com mais esta?

*Sendo |z|<1/2 e fazendo uso da desigualdade triangular, |z1 + z2| < |z1| +
|z2|, mostre que:*
*|(1 + i)z³ + iz| < 3/4*

Agradeço antecipadamente.
Ney



=
Instruções para entrar na lista, sair da lista e usar a lista em
http://www.mat.puc-rio.br/~obmlistas/obm-l.html
=


Re: [obm-l] Matrizes

2007-11-23 Por tôpico LEANDRO L RECOVA
A pergunta foi muito geral. O que voce quer calcular? Determinantes? 
Multiplicacao de matrizes? Resolucao de sistemas lineares? Autovalores?


leandro





From: "Bruno França dos Reis" <[EMAIL PROTECTED]>
Reply-To: obm-l@mat.puc-rio.br
To: obm-l@mat.puc-rio.br
Subject: Re: [obm-l] Matrizes
Date: Fri, 23 Nov 2007 21:33:59 +0100

Ola.

Eu lembro de ter estuda um pouco um livro de uma coleção de 2 volumes, acho
que o autor chama-se Gantmacher. Eu achei muito muito bom.

Bruno

2007/11/23, nexthere <[EMAIL PROTECTED]>:
>
> Existe algum método mais rápido de calcular matrizes que não seja por
> esses métodos mais usuais que aprendemos no ensino médio? Se tem alguém 
pode

> ensinar-me?
>
> Atenciosamente,
>
> César Augusto.
>



--
Bruno FRANÇA DOS REIS

msn: [EMAIL PROTECTED]
skype: brunoreis666
tel: +33 (0)6 28 43 42 16

e^(pi*i)+1=0



=
Instruções para entrar na lista, sair da lista e usar a lista em
http://www.mat.puc-rio.br/~obmlistas/obm-l.html
=


Re: [obm-l] Autovalor

2007-11-17 Por tôpico LEANDRO L RECOVA


Marcelo,

Obrigado pela observacao, mas eu considerei que na pergunta do Klauss a 
matriz fosse diagonalizavel ja que ele queria mostrar a formula do 
determinante de A igual ao produto dos autovalores. Esqueci de supor que ela 
era diagonalizavel, falha minha. Voce esta certo e sua abordagem foi 
excelente.


Valeu,

Leandro.


From: "Marcelo Salhab Brogliato" <[EMAIL PROTECTED]>
Reply-To: obm-l@mat.puc-rio.br
To: obm-l@mat.puc-rio.br
Subject: Re: [obm-l] Autovalor
Date: Fri, 16 Nov 2007 19:02:59 -0200

só li o email original agora..

Klaus, para provar que o traco é igual a soma dos autovalores, veja que o
coeficiente de a^(n-1), em modulo, é igual a traço(A)...
e que o sinal é sempre oposto ao coeficiente de a^n...
e use a idéia do polinomio que mandei no outro email.

abraços,
Salhab


On Nov 16, 2007 7:00 PM, Marcelo Salhab Brogliato <[EMAIL PROTECTED]>
wrote:

> Olá Leandro,
>
> nem toda matriz quadrada pode ser escrita como A = PSP^-1...
> uma outra possível abordagem seria:
> dizemo que "k" é autovalor quando: P(k) = det(A - kI) = 0
> fazendo k=0, temos que P(0) = det(A)
> no polinomio, quando k=0, temos apenas o termo independente...
> sabemos que o coeficiente do termo de maior grau é 1 quando n (dimensao 
de

> A) é par e -1 quando n é impar..
> as raizes do polinomio sao os autovetores..
> sabemos que P(0) = (-1)^n.k1.k2...kn / (-1)^n = k1.k2.k3...kn
> logo: det(A) = k1.k2.k3...kn
>
> abraços,
> Salhab
>
>
>
>
> On Nov 16, 2007 2:54 PM, LEANDRO L RECOVA < [EMAIL PROTECTED]> 
wrote:

>
> > Klauss,
> >
> >
> > Na ultima pergunta, se voce supor a matriz quadrada, lembre que voce
> > pode
> > decompo-la na forma A=PSP^-1, onde P e a matriz cujas colunas contem 
os
> > autovetores de A e S e a matriz diagonal com os autovalores de A.  
Segue
> > imediato que o det(A)=det(S)=produto dos autovalores de A. Agora o 
traco

> > e
> > facil de calcular e deixo pra voce.
> >
> > Regards,
> >
> > Leandro
> > Los Angeles, CA.
> >
> >
> > >From: Klaus Ferraz <[EMAIL PROTECTED]>
> > >Reply-To: obm-l@mat.puc-rio.br
> > >To: obm-l@mat.puc-rio.br
> > >Subject: [obm-l] Autovalor
> > >Date: Tue, 13 Nov 2007 17:09:42 -0800 (PST)
> > >
> > >Dado A E R n x n
> > >Se A= A^T então todo autovalor de A Ã(c) real
> > >Se A=-A^T então todo autovalor de Ã(c) da forma ir, r E R
> > >
> > >TambÃ(c)m como que eu mostro que o produto dos autovalores de uma 
matriz

> > Ã(c)
> > >igual ao seu determinante e o traço igual a soma dos autovalores.
> > >Grato.
> > >
> > >
> > >   Abra sua conta no Yahoo! Mail, o único sem limite de espaço
> > para
> > >armazenamento!
> > > http://br.mail.yahoo.com/
> >
> >
> >
> > 
=

> > Instruções para entrar na lista, sair da lista e usar a lista em
> > 
http://www.mat.puc-rio.br/~obmlistas/obm-l.html<http://www.mat.puc-rio.br/%7Eobmlistas/obm-l.html>

> >
> > 
=

> >
>
>



=
Instruções para entrar na lista, sair da lista e usar a lista em
http://www.mat.puc-rio.br/~obmlistas/obm-l.html
=


RE: [obm-l] Autovalor

2007-11-16 Por tôpico LEANDRO L RECOVA

Klauss,


Na ultima pergunta, se voce supor a matriz quadrada, lembre que voce pode 
decompo-la na forma A=PSP^-1, onde P e a matriz cujas colunas contem os 
autovetores de A e S e a matriz diagonal com os autovalores de A.  Segue 
imediato que o det(A)=det(S)=produto dos autovalores de A. Agora o traco e 
facil de calcular e deixo pra voce.


Regards,

Leandro
Los Angeles, CA.



From: Klaus Ferraz <[EMAIL PROTECTED]>
Reply-To: obm-l@mat.puc-rio.br
To: obm-l@mat.puc-rio.br
Subject: [obm-l] Autovalor
Date: Tue, 13 Nov 2007 17:09:42 -0800 (PST)

Dado A E R n x n
Se A= A^T então todo autovalor de A é real
Se A=-A^T então todo autovalor de é da forma ir, r E R

Também como que eu mostro que o produto dos autovalores de uma matriz é 
igual ao seu determinante e o traço igual a soma dos autovalores.

Grato.


  Abra sua conta no Yahoo! Mail, o único sem limite de espaço para 
armazenamento!

http://br.mail.yahoo.com/



=
Instruções para entrar na lista, sair da lista e usar a lista em
http://www.mat.puc-rio.br/~obmlistas/obm-l.html
=


RE: [obm-l] Pot�ncia Complexa

2007-11-02 Por tôpico LEANDRO L RECOVA

Carissimo Ricardo,

Use o fato de que 2^(ix) = e^(ln(2^ix))=e^(i(x*ln(2))

Lembre que e^(ibx)=cos(bx)+isin(bx) , entao, fazendo b=x*ln(2), temos

2^(ix) = cos(xln(2))+isin(xln(2)).

No caso geral, em que z e w sao complexos e voce quer calcular z^w, use o 
mesmo raciocinio e nao esqueca que voce vai necessitar definir um ramo da 
funcao logaritmo, pois nesse caso a funcao z^w e multivalente. Defina 
z=p(cos(theta)+isin(theta)), p > 0,


z^w=e^(log(z^w)) = e^(w*log(z)) = e^(w*(ln|p|+i.theta)). Para que isso tenha 
sentido, p > 0 e theta pode ser , por exemplo, o intervalo ]-pi,pi[.


Isso nao e nada formal o que escrevi, e mais detalhes voce pode encontrar no 
livro do Churchill (Variaveis Complexas) ou do Alfhors (Complex Analysis).


Leandro.


From: Ricardo Khawge <[EMAIL PROTECTED]>
Reply-To: obm-l@mat.puc-rio.br
To: 
Subject: [obm-l] Potência Complexa
Date: Fri, 2 Nov 2007 02:53:03 +0300


Gostaria de uma ajuda para aprender a determinar o valor de a^(x+bi). Por 
exemplo, sei desenvolver em série de Taylor 2^ix e sei que  e^ix=cos x+ 
isenx. Com juntar isso para calcular 2^i, 2^ix ou 2^(x+bi) sem usar série?

Não consigo obter 2^ix = cos(xln2) + i sen(xln2)
Obrigado


_
Conheça o Windows Live Spaces, a rede de relacionamentos conectada ao 
Messenger!

http://spaces.live.com/signup.aspx



=
Instruções para entrar na lista, sair da lista e usar a lista em
http://www.mat.puc-rio.br/~obmlistas/obm-l.html
=


[obm-l] 2^k=k^2

2007-11-01 Por tôpico leandro oliveira

Na verdade, ninguem conseguiu resolver esse desafio, pois existe uma terceira 
resposta que é aproximadamente -0,767. Mas se bem que é meio difícil de 
testá-la, pois a consegui com um programa que constroi gráficos e construi o 
gráfico de 2^k-k^2=y, achando então 3 soluções.
Espero que alguem consiga resolver o exercício em questão e achar a resposta 
precisa, sem programas, sem chutes.
_
Receba GRÁTIS as mensagens do Messenger no seu celular quando você estiver 
offline. Conheça  o MSN Mobile!
http://mobile.live.com/signup/signup2.aspx?lc=pt-br

Re: [obm-l] Integral

2007-10-15 Por tôpico LEANDRO L RECOVA

Vivian,

Tens razao, devia ter feito uma substituicao diferente. Nao estava com lapis 
e papel do lado. Agora arranjei um aqui e fiz. No seu resultado, nao sei se 
voce quis dizer arco-tangente ou arco-cotangente. A minha integral coincide 
com a sua se considerar o arco-cotangente e eu a derivei essa vez e esta 
correta agora.


Olha, quando voce ver potencias de x ao quadrado, por exemplo, x^2+4, 1-x^2, 
etc, tente construir um triangulo retangulo e coloque nos catetos por 
exemplo, no seu caso, o cateto oposto como a variavel sqrt(2), o cateto 
adjacente a variavel x, o angulo entre a hipotenusa e cateto adjancente voce 
chama de t, e a hipotenusa sera sqrt(x^2+2). Isso e o que chamei de 
substituicao trigonometrica. Nao foi magica como o nosso amigo anterior 
falou e nem arte, e um artificio matematico que todo professor de calculo 
ensina os estudantes a fazer.


Voltando ao problema,

sin(t)=sqrt(2/x^2+2)  (Faca o triangulo retangulo como eu disse).
x=sqrt(2)cotg(t)  (Confira no triangulo retangulo)

=> dx=-sqrt(2)cosec^2(t)

1/(x^2+2)^2 = sin^4(t)/4

Entao,

I = int (sin^4(t)/4)*(-sqrt(2)cosec^2(t))dt

I = -sqrt(2)*int(sin^2(t))/4 dt

I = -(sqrt(2)/4) * int (1/2 - cos(2t)/2)dt

I = -(sqrt(2)/8) * [t - sin(2t)/2] + C

Lembre que sin(2t)=2*cost(2)*sin(t)=2*(sqrt(2/x^2+2)*(x/x^2+2); entao,


I = -(1/4*sqrt(2))*[actg(x/sqrt(2)) - (sqrt(2).x)/(x^2+2)] + C

I = x/(4*(x^2+2)) - (1/4*sqrt(2))*arccotg(x/sqrt(2));


Lembre-se que 1-sin^2(t)=cos(2t) => sin^2(t)=1/2-cos(2t)/2


Saudacoes,

Leandro
Los Angeles, CA.



From: "Vivian Heinrichs" <[EMAIL PROTECTED]>
Reply-To: obm-l@mat.puc-rio.br
To: obm-l@mat.puc-rio.br
Subject: Re: [obm-l] Integral
Date: Fri, 12 Oct 2007 21:28:42 -0300

Desculpe minha ignorância, mas o que é sqrt?
Em um livro vi que a resposta da Integral I = dx/(x^2 + 2)^2 é igual a
(x/4(x^2 + 2)) + 1/(4*2^1/2) * arctg (x/(2*1/2)) + C, sendo C a 
constante...

Não cosigo chegar a esta resposta... e por minha ignorância não cosegui
entender a resolução proposta...
Se alguém coseguir me ajudar, agradeço...
Muito Obrigada.


Em 12/10/07, LEANDRO L RECOVA <[EMAIL PROTECTED]> escreveu:
>
> Voce pode usar a seguinte substituicao trigonometrica:
>
> (1) sin(t)=sqrt(2)/(x^2+2)
>
> (2) x=sqrt(2).cotg(t)
>
> Entao, de (2) temos:
>
> dx=-sqrt(2)cosec^2(t)
>
> Substituindo na integral temos,
>
> I = int [ -sqrt(2)*csc^2(t)/(2/sin^2(t)]dt
>
> I = int [-sqrt(2)/2]dt
>
> I = [-sqrt(2)/2]*t + C,  C e uma constante de integracao. Substituindo 
(1)

> nessa equacao temos
>
> I = [-sqrt(2)/2]*arcsin(2/(x^2+2)) + C
>
> Saudacoes rubro-negras,
>
> Leandro
> Los Angeles, CA.
>
> >From: "Vivian Heinrichs" <[EMAIL PROTECTED]>
> >Reply-To: obm-l@mat.puc-rio.br
> >To: obm-l@mat.puc-rio.br
> >Subject: [obm-l] Integral
> >Date: Fri, 12 Oct 2007 13:30:33 -0300
> >
> >Olá pessoal...
> >Gostaria de saber se alguém sabe resolver a Integral : I = dx/(x^2 + 
2)^2

> ,
> >sendo que I é a Integral.
> >Obrigada.
>
>
> 
=

> Instruções para entrar na lista, sair da lista e usar a lista em
> http://www.mat.puc-rio.br/~obmlistas/obm-l.html
> 
=

>



=
Instruções para entrar na lista, sair da lista e usar a lista em
http://www.mat.puc-rio.br/~obmlistas/obm-l.html
=


RE: [obm-l] Integral

2007-10-12 Por tôpico LEANDRO L RECOVA

Voce pode usar a seguinte substituicao trigonometrica:

(1) sin(t)=sqrt(2)/(x^2+2)

(2) x=sqrt(2).cotg(t)

Entao, de (2) temos:

dx=-sqrt(2)cosec^2(t)

Substituindo na integral temos,

I = int [ -sqrt(2)*csc^2(t)/(2/sin^2(t)]dt

I = int [-sqrt(2)/2]dt

I = [-sqrt(2)/2]*t + C,  C e uma constante de integracao. Substituindo (1) 
nessa equacao temos


I = [-sqrt(2)/2]*arcsin(2/(x^2+2)) + C

Saudacoes rubro-negras,

Leandro
Los Angeles, CA.


From: "Vivian Heinrichs" <[EMAIL PROTECTED]>
Reply-To: obm-l@mat.puc-rio.br
To: obm-l@mat.puc-rio.br
Subject: [obm-l] Integral
Date: Fri, 12 Oct 2007 13:30:33 -0300

Olá pessoal...
Gostaria de saber se alguém sabe resolver a Integral : I = dx/(x^2 + 2)^2 ,
sendo que I é a Integral.
Obrigada.



=
Instruções para entrar na lista, sair da lista e usar a lista em
http://www.mat.puc-rio.br/~obmlistas/obm-l.html
=


RE: [obm-l] GEN�SIO

2007-10-11 Por tôpico LEANDRO L RECOVA

X - Variavel aleatoria: Meio de transporte.
Y - Variavel aleatoria: Chegou ao congresso com dois dias de atraso.

P1=P(X=Navio) = 0.4
P2=P(X=Aviao) = 0.6

P3=P(Y=Congresso, X=Navio) = 0.085
P4=P(Y=Congresso, X=Aviao) = 0.01

P5=P(Y=Congresso) = P3*P1 + P4*P2 = 0.04
P6=(Y=Congresso e X=aviao) = P2*P4 = 0.006

Entao, temos que a probabilidade de Genesio ter ido de aviao e dado por: 
(Teorema de Bayes)


P7=P(X=aviao) = P6/P5 = 0.15.

E a de ter ido de Navio e dada por 1-P7=0.85.

Regards,

Leandro.
Los Angeles, CA.


From: "arkon" <[EMAIL PROTECTED]>
Reply-To: obm-l@mat.puc-rio.br
To: "obm-l" 
Subject: [obm-l] GENÉSIO
Date: Thu, 11 Oct 2007 16:32:50 -0300

Alguém pode resolver, por favor, esta:
(ESAF) Há apenas dois modos, mutuamente excludentes, de Genésio ir para 
Genebra participar de um congresso: ou de navio ou de avião. A 
probabilidade de Genésio ir de navio é de 40% e de ir de avião é de 60%. Se 
ele for de navio, a probabilidade de chegar ao congresso com dois dias de 
atraso é de 8,5%. Se ele for de avião a probabilidade de chegar ao 
congresso com dois dias de atraso é de 1%. Sabe-se que Genésio chegou com 
dois dias de atraso para participar do congresso em Genebra. A 
probabilidade de ele ter ido de avião é:

a) 5%.  b) 8%.  c) 10%.  d) 15%.   e) 18%.

GABARITO LETRA D
DESDE JÁ AGRADEÇO



=
Instruções para entrar na lista, sair da lista e usar a lista em
http://www.mat.puc-rio.br/~obmlistas/obm-l.html
=


RE: RES: [obm-l] integral

2007-10-10 Por tôpico LEANDRO L RECOVA

Coloque o denominador na forma (t-sqrt(2)/2)^2

Entao, sua integral fica facil de resolver.

I = int [t/(t-sqrt(2))^2] dt

Chame z=t-sqrt(2) => dz=dt, t=z+sqrt(2), entao,

I = int [ (z+sqrt(2))/z^2] dz

I = int [ 1/z + sqrt(2)/z^2] dz

I = ln(z) - sqrt(2)/z + C, onde C e uma constante de integracao.

Agora, substitua z=t-sqrt(2)

I = ln(t-sqrt(2)) - sqrt(2)/(t-sqrt(2)) + C.

Lembre que esse problema tem solucao somente se t > sqrt(2).

Se nao errei em sinal, essa deve ser a solucao.

Regards,

Leandro
Los Angeles, CA.





From: "Marcus" <[EMAIL PROTECTED]>
Reply-To: obm-l@mat.puc-rio.br
To: 
Subject: RES: [obm-l] integral
Date: Tue, 9 Oct 2007 22:44:31 -0300

Poxa gente desculpem mas coloquei um sinal errado. Na verdade era
int (tdt) / (1 - sqrt(2)t + t^2)



-Mensagem original-
De: [EMAIL PROTECTED] [mailto:[EMAIL PROTECTED] Em nome
de Arlane Manoel S Silva
Enviada em: terça-feira, 9 de outubro de 2007 12:43
Para: obm-l@mat.puc-rio.br
Assunto: Re: [obm-l] integral

Basta notar que
int (tdt) / (1 - sqrt(2)t - t^2) = int {-t/[(t-t1)(t-t2)]}dt,
   onde
  t1= -[sqrt(2)+sqrt(6)]/2
e
  t2= -[sqrt(2)-sqrt(6)]/2
  daí é só resolver através de frações parciais...


Citando Marcus <[EMAIL PROTECTED]>:

> Alguém tem uma idéia para resolver esta integral...integral de (tdt) / 1 
-

> sqrt(2)t - t^2
>
>
>
> Marcus Aurélio
>
>
>
>



--
Arlane Manoel S Silva
   MAT-IME-USP


=
Instruções para entrar na lista, sair da lista e usar a lista em
http://www.mat.puc-rio.br/~nicolau/olimp/obm-l.html
=



=
Instruções para entrar na lista, sair da lista e usar a lista em
http://www.mat.puc-rio.br/~nicolau/olimp/obm-l.html
=



=
Instruções para entrar na lista, sair da lista e usar a lista em
http://www.mat.puc-rio.br/~nicolau/olimp/obm-l.html
=


RE: [obm-l] Equação diferencial

2007-10-02 Por tôpico LEANDRO L RECOVA

Daniel,

Tente a substituicao: z=y/x

y' = (xz)' = z'x + z

Entao, sua EDO fica

z'x + z = z + sqrt(x^2.z)

x.z' = x.sqrt(z)  (Coloque na forma separavel)

(dz/sqrt(z)) = dx, Integre ambos os lados

2.sqrt(z) = x + A ,

sqrt(z) = x/2 + C, C=A/2

z = (x/2 + C)^2 , faca a substituicao z=y/x,

y = x.[(x/2) + C)]^2 , onde C e uma constante de integracao.

Se fiz algum erro nas contas, me perdoe. Nao tinha papel aqui e tentei fazer 
direto no computador.



Leandro
Los Angeles, CA.


From: "Daniel S. Braz" <[EMAIL PROTECTED]>
Reply-To: obm-l@mat.puc-rio.br
To: OBM-L 
Subject: [obm-l] Equação diferencial
Date: Sat, 22 Sep 2007 12:36:19 -0300

Senhores,
Alguém pode, por favor, me dar uma dica nessa eq. diferencial?
dy/dx = y/x + sqrt(xy)
obrigado.
Daniel.
-- "O modo mais provável do mundo ser destruído, como concordam a 
maioriados especialistas, é através de um acidente. É aí que nós 
entramos.Somos profissionais da computação. Nós causamos acidentes" - 
NathanielBorenstein

=
Instrues para entrar na lista, sair da lista e usar a lista em
http://www.mat.puc-rio.br/~nicolau/olimp/obm-l.html
=



=
Instruções para entrar na lista, sair da lista e usar a lista em
http://www.mat.puc-rio.br/~nicolau/olimp/obm-l.html
=


RE: [obm-l] transforma�

2007-10-02 Por tôpico LEANDRO L RECOVA

E so voce lembrar que:

cos(4x) = cos^(2x) - sin^2(2x)
cos(4x) = 1-2sin^2(2x)

sin^2(2x) = (1- cos(4x))/2

entao, sua transformacao esta correta.



From: "Marcus" <[EMAIL PROTECTED]>
Reply-To: obm-l@mat.puc-rio.br
To: 
Subject: [obm-l] transformação
Date: Tue, 2 Oct 2007 13:53:16 -0300

Algum pode me dizer se a transformação que to fazendo tem algum erro:

Sen^2(2x)= (1- cos4x)/2



Marcus Aurélio






=
Instruções para entrar na lista, sair da lista e usar a lista em
http://www.mat.puc-rio.br/~nicolau/olimp/obm-l.html
=


RE: [obm-l] Uma pequena luz nestas quest

2007-09-20 Por tôpico LEANDRO L RECOVA


1) Na primeira questao, voce deve estabelecer que os dois graficos para 
serem tangentes em (0,0) devem ter a mesma reta tangente pertencente aos 
dois graficos. Se a equacao da reta y=kx nesse ponto, entao devemos ter:


k=f'(0)=2a(0) + b = b

k=g'(0) = 2c(0) + d = d


Devemos ter b=d em uma das igualdades.  Devemos ter f(0)=0 e g(0)=0. Da 
ultima igualdade voce tira que e=0.  a e b devem ser diferentes de zero para 
que as parabolas existam.



2) Para que a reta seja tangente paralela ao eixo x, o seu coeficiente 
angular deve ser zero. Entao, a reta tangente ao ponto (x1,y1) do grafico de 
f tem coeficiente angular dado por:


f'(x1) = 3(x1)^2 + b

Como queremos encontrar f'(x1)=0, entao, deveriamos ter x1 = (+/-) 
sqrt(-b/9). Como b > 0, entao, x1 nao e um numero real, e, portanto, nao 
podemos obter uma reta tangente ao grafico de f(x) para b > 0.


Leandro
Los Angeles, CA.


From: "vitoriogauss" <[EMAIL PROTECTED]>
Reply-To: obm-l@mat.puc-rio.br
To: "obm-l" 
Subject: [obm-l] Uma pequena luz nestas questões
Date: Thu, 20 Sep 2007 16:02:11 -0300

 01) Como devem ser os números a, b, c, d e e para que os gráficos de 
f(x)=ax^2+bx e g(x)=cx^2+dx+e, sejam tangentes em (0,0).



02) Mostre que, se b>0, não existem tangentes ao gráfico de f(x)=x^3+bx+c 
que são paralelas ao eixo dos x.



abraços



=
Instruções para entrar na lista, sair da lista e usar a lista em
http://www.mat.puc-rio.br/~nicolau/olimp/obm-l.html
=


Re: [obm-l] Derivada parcial

2007-09-05 Por tôpico LEANDRO L RECOVA
Acho que seu conceito de derivada total como "soma das derivadas parciais" 
nao esta correto. Por favor, para uma definicao correta, olhe qualquer livro 
de calculo avancado ou no Mathworld ou wilkipedia,


http://mathworld.wolfram.com/TotalDerivative.html.

http://en.wikipedia.org/wiki/Total_derivative

Para o calculo da derivada total, iriamos precisar saber a direcao em que 
estamos procurando essa derivada como o amigo Alonso mencionou em seu email 
anterior. A questao esta muito aberta. Nao sabemos se x e y sao 
independentes, ou se y=f(x), etc. Pediria ao colega para colocar o enunciado 
correto da questao.



Regards,

Leandro Recova.
Los Angeles, CA.



From: johnson nascimento <[EMAIL PROTECTED]>
Reply-To: obm-l@mat.puc-rio.br
To: obm-l@mat.puc-rio.br
Subject: Re: [obm-l] Derivada parcial
Date: Wed, 5 Sep 2007 11:11:09 -0300 (ART)

Giovani a derivada total é a soma das derivadas parcais.
  Isso significa que voçe ira ter que fazer umas constante e derivar em 
relação a que voçe considerou variavel, e assim sucessivamente.

  Exemplo :
  derivar em relação a x "dz/dx" voçe irar trandormar z = xe^(x - y) + 
ye^(x + y) em  z = xe^(x-a) + ae^(x+a)
  derivar em relação a y "dz/dy" irar ter que transformar z = xe^(x - y) + 
ye^(x + y). em z = ae^(a-y) + ye^(a+y).

  E realizar as derivações.

lembrando que a derivada total é a soma das derivadas parciais.

espero ter te ajudado.


giovani ferrera <[EMAIL PROTECTED]> escreveu:


Ola... por favor, como derivar essa?
z = xe^(x - y) + ye^(x + y).

_
Inscreva-se no novo Windows Live Mail beta e seja um dos primeiros a testar
as novidades-grátis. Saiba mais:
http://www.ideas.live.com/programpage.aspx?versionId=5d21c51a-b161-4314-9b0e-4911fb2b2e6d

=
Instruções para entrar na lista, sair da lista e usar a lista em
http://www.mat.puc-rio.br/~nicolau/olimp/obm-l.html
=


   Flickr agora em português. Você clica, todo mundo vê. Saiba mais.



=
Instruções para entrar na lista, sair da lista e usar a lista em
http://www.mat.puc-rio.br/~nicolau/olimp/obm-l.html
=


[obm-l] Dúvidas

2007-09-03 Por tôpico Leandro Morelato
Pessoal tenho o seguinte exercício:



Encontre os quatro primeiros elementos da seqüência de somas parciais {Sn},
e obtenha uma fórmula para *Sn*, em termos de *n*. Determine também se a
série infinita é convergente ou divergente; se for convergente, encontre a
sua soma.



+∞

Σ ln(n/n+1)

n=1



Resposta: Sn = -ln(n+1); diverge



Encontrar os quatro primeiros elementos é simples, fazendo uma análise
também da para descobrir que é divergente, agora a formula de Sn em termos
de n, não consegui. Podem me ajudar?



Leandro


RE: [obm-l] off topic: algebra linear

2007-08-06 Por tôpico LEANDRO L RECOVA


Tente um livro da Colecao Schaum do Lipchultz.

Tem muito exercicio resolvido.


From: "Tio Cabri st" <[EMAIL PROTECTED]>
Reply-To: obm-l@mat.puc-rio.br
To: 
Subject: [obm-l] off topic: algebra linear
Date: Wed, 1 Aug 2007 15:40:26 -0300

Senhores boa tarde, preciso de uma LUZ ou melhor uma grande luz.

Precisei reestudar (se é que um dia eu já aprendi!?)
álgebra linear e me deparei com dois problemas:

i) cada livro possui um sumário diferente com ordens bem disdintas um do
outro. Confunde.
ii) os famosos 'se vire nos exercícios sem respostas para complementar o
entendimento do capítulo' deixando aqueles que estão estudando sozinhos
completamente frustrados.

Uma grande amiga que frequenta a lista me arrumou uns exercícios do CEDERJ.

Peço a gentileza de me indicarem um livro que possua mais exercícios
resolvidos ou se possível
algum material com problemas e respostas.
Obrigado

Atenciosamente, Tio Cabri

[EMAIL PROTECTED]

=
Instruções para entrar na lista, sair da lista e usar a lista em
http://www.mat.puc-rio.br/~nicolau/olimp/obm-l.html
=



=
Instruções para entrar na lista, sair da lista e usar a lista em
http://www.mat.puc-rio.br/~nicolau/olimp/obm-l.html
=


RE: [obm-l] fun��o lipschitz

2007-07-28 Por tôpico LEANDRO L RECOVA

=> f e funcao de Lipchitz, entao, existe C > 0 tal que , para x,y em I temos

   |f(x)-f(y)| =< c . |x-y|

Portanto, |(f(x)-f(y))/(x-y)| =< c, o que prova que f' e limitada.

<= A volta e imediata. Supondo f' limitada, entao, existe c > 0 tal que 
|(f(x)-f(y))/(x-y)| =< c , entao, |f(x)-f(y)| =< c . |x-y|. f e Lipchitz.


Alem disso, f e uniformemente continua tambem!

Regards,

Leandro Recova
Los Angeles, CA.



From: "Kleber Bastos" <[EMAIL PROTECTED]>
Reply-To: obm-l@mat.puc-rio.br
To: obm-l@mat.puc-rio.br
Subject: [obm-l] função lipschitz
Date: Fri, 27 Jul 2007 21:30:54 -0300

Poderiam me ajudar ?

Mostre que f :I-->R, onde I C R  é um intervalo é uma função Lipschitz se ,
e smomente se f ´ ( f linha )  é uma função limitada em I .

--
Kleber B. Bastos



=
Instruções para entrar na lista, sair da lista e usar a lista em
http://www.mat.puc-rio.br/~nicolau/olimp/obm-l.html
=


RE: [obm-l] Re: Integral sin(x)/x

2007-06-12 Por tôpico LEANDRO L RECOVA

Hugo,

Essa funcao e muito estudada, por exemplo, num curso de comunicacoes 
digitais quando voce estuda alguns tipos de modulacoes digitais. Seria 
off-topic falar aqui, e se quiser mais detalhes, me mande um email. Esta 
relacionada tambem com o Teorema de Nyquist para determinar a taxa de 
amostragem de um sinal para que nao haja sobreposicao do sinal.  Dependendo 
do valor da taxa de amostragem, voce pode reconstruir um sinal somente com 
as amostras que voce obteve no receptor.


Leandro
Los Angeles, CA.



From: "Hugo Canalli" <[EMAIL PROTECTED]>
Reply-To: obm-l@mat.puc-rio.br
To: obm-l@mat.puc-rio.br
Subject: [obm-l] Re: Integral sin(x)/x
Date: Sun, 10 Jun 2007 21:16:57 -0300

Expliquei errado, a integral a gente calcula a partir da série de Taylor...
Então o que queria saber era:
Sendo,
lim (sen(t)/t)dt  = 1
t-->0
O que se poderia afirmar sobre a derivada primeira ou segunda da função
(sen(t)/t)dt ?


Em 09/06/07, Hugo Canalli <[EMAIL PROTECTED]> escreveu:


Pessoal, o que de especial a função sin(x)/x? Bem, significa que ela não
elementar...

Então como ficaria a integral de sin(x)/x de 0 até um x (definido ou
não)?. O que ajudaria saber que:

lim (sen(t)/t)dt  = 1
t-->0

--
[]'s





=
Instruções para entrar na lista, sair da lista e usar a lista em
http://www.mat.puc-rio.br/~nicolau/olimp/obm-l.html
=


  1   2   3   4   5   >